GENERAL PRACTITIONER EXAM
Question Summary
0 of 100 questions completed
Questions:
- 1
- 2
- 3
- 4
- 5
- 6
- 7
- 8
- 9
- 10
- 11
- 12
- 13
- 14
- 15
- 16
- 17
- 18
- 19
- 20
- 21
- 22
- 23
- 24
- 25
- 26
- 27
- 28
- 29
- 30
- 31
- 32
- 33
- 34
- 35
- 36
- 37
- 38
- 39
- 40
- 41
- 42
- 43
- 44
- 45
- 46
- 47
- 48
- 49
- 50
- 51
- 52
- 53
- 54
- 55
- 56
- 57
- 58
- 59
- 60
- 61
- 62
- 63
- 64
- 65
- 66
- 67
- 68
- 69
- 70
- 71
- 72
- 73
- 74
- 75
- 76
- 77
- 78
- 79
- 80
- 81
- 82
- 83
- 84
- 85
- 86
- 87
- 88
- 89
- 90
- 91
- 92
- 93
- 94
- 95
- 96
- 97
- 98
- 99
- 100
Information
Hi, Welcome to GENERAL PRACTITIONER EXAM
You have already completed the Exam before. Hence you can not start it again.
Exam is loading...
You must sign in or sign up to start the Exam.
You have to finish following Exam, to start this Exam:
Results
0 of 100 questions answered correctly
Your time:
Time has elapsed
You have reached 0 of 0 points, (0)
Average score |
|
Your score |
|
Categories
- Not categorized 0%
Pos. | Name | Entered on | Points | Result |
---|---|---|---|---|
Table is loading | ||||
No data available | ||||
- 1
- 2
- 3
- 4
- 5
- 6
- 7
- 8
- 9
- 10
- 11
- 12
- 13
- 14
- 15
- 16
- 17
- 18
- 19
- 20
- 21
- 22
- 23
- 24
- 25
- 26
- 27
- 28
- 29
- 30
- 31
- 32
- 33
- 34
- 35
- 36
- 37
- 38
- 39
- 40
- 41
- 42
- 43
- 44
- 45
- 46
- 47
- 48
- 49
- 50
- 51
- 52
- 53
- 54
- 55
- 56
- 57
- 58
- 59
- 60
- 61
- 62
- 63
- 64
- 65
- 66
- 67
- 68
- 69
- 70
- 71
- 72
- 73
- 74
- 75
- 76
- 77
- 78
- 79
- 80
- 81
- 82
- 83
- 84
- 85
- 86
- 87
- 88
- 89
- 90
- 91
- 92
- 93
- 94
- 95
- 96
- 97
- 98
- 99
- 100
- Unanswered
- Answered
- Review
-
Question 1 of 100
1. Question
1 pointsA 5 year old complains of a sore throat and fever for 2 weeks. There is cervical adenopathy and a sand paper like rash on physical exam. A diagnosis of scarlet fever is made. What investigation will differentiate this from a viral etiology?
Correct
Incorrect
Explanation:
Scarlet fever is an illness that brings on a rash covering most of the body, a strawberry-like appearance of the tongue and usually a high fever. The most common source of scarlet fever is one form of a common bacterial infection known as strep throat.
Common symptoms are: Red rash that looks like a sunburn and feels like sandpaper, strawberry-like red and bumpy appearance of the tongue. Fever of 101 degrees Fahrenheit or higher, often with chills. Very sore and red throat sometimes with white or yellowish patches. Enlarged glands in the neck (lymph nodes) that are tender.
A bacterium called Streptococcus pyogenes, or group A beta-hemolytic streptococcus causes scarlet fever. Throat culture remains the criterion standard for confirmation of group A streptococcal upper respiratory infection.
Streptococcal antibody tests are used to confirm previous group A streptococcal infection. The most commonly available streptococcal antibody test is the Antistreptolysin O test (ASO). Currently, streptococcal antibody tests are not indicated during acute illness. Possible complications include rheumatoid fever, and poststreptococcal glomerulonephritis. Treatment is with antibiotics such as penicillin and amoxicillin. -
Question 2 of 100
2. Question
1 pointsA 2 year old child has fever seen following a generalized seizure. He also has foul smelling, blood-tinged diarrhea. The culture of the stool is most likely to grow
Correct
Incorrect
Explanation:
Shigellosis is infection with species of the gram-negative bacillus Shigella, which results in dysentery that is characterized by frequent watery stools, often with mucus and blood, pain, fever, and dehydration.
Fever is frequently present and may reach 106˚F. Some children develop seizures. It is not known if these seizures occur simply from the high fever or as a specific complication of shigellosis. -
Question 3 of 100
3. Question
1 pointsIn a 1 year old, pneumococcal 7-valent conjugate vaccine (prevnar) is preferred rather than polyvalent pneumococcal vaccine (pneumovax) because of which one of the following advantages?
Correct
Incorrect
Explanation:
Pneumococcal 7-valent vaccine produces a satisfactory immune response in a 1 year old, while polyvalent vaccine does not cause a good antibody response in children under the age of 2. Neither vaccine is available orally, nor is cost not a factor. The 7-valent vaccine requires multiple doses. Neither vaccine can be combined with MMR.
-
Question 4 of 100
4. Question
1 pointsWhich one of the following is the most common cause of infectious enteritis in child in temperate climates?
Correct
Incorrect
Explanation:
Rotavirus is the most common agent responsible for infantile diarrhea throughout the world in contrast to Vibrio. In tropical climates, rotavirus is the etiologic agent in 15%-50% of cases of acute diarrhea in children, and in temperate climates it accounts for 35%-60% of cases. Cytomegaolvirus has been shown to infect the colon on occasion. Bacterial pathogens account for 10%-15% of cases of acute childhood diarrhea: the most commonly identified bacterial pathogens in North American children are Campylobacter jejuni and various strains of salmonella and Shigella species. Some strains of Escherichia coli are pathogenic, causing sporadic cases of acute enteritis, epidemic diarrhea, and traveler´s diarrhea.
-
Question 5 of 100
5. Question
1 pointsA 5 year old child accidently steps on a rusty nail. He then visits the emergency room. History reveals that he has his DTaP vaccinations at 2, 4, 6 months and 18 months of age and a booster DT vaccination one year ago. Which of the following would you recommend?
Correct
Incorrect
Explanation:
There are four combination vaccines used to prevent diphtheria, tetanus and pertussis: DTaP, Tdap, DT, and Td. Two of these (DTaP and DT) are given top children younger than 7 years of age, and two (Tdap and Td) are given to older children and adults. Children should get 5 doses of DTaP, one dose at each of the following ages: 2, 4, 6, and 15-18 month´s andr4-6 years. DT does not contain pertussis and is used as a substitute for DTaP for children who cannot tolerate pertussis vaccine. Td is a tetanus-diphtheria vaccine given to adolescents and adults as a booster shot every 10 years, or after an exposure to tetanus under some circumstances. Tdap is similar to Td but also containing protection against pertussis. A single dose of Tdap is recommended for adolescents 11 or 12 years of age, or in place of one Td booster in older adolescents and adults age 19 through 64.
-
Question 6 of 100
6. Question
1 pointsA 6 year old boy develops a rash. Koplik´s spots can also be seen. Fever and cough are associated with it. He is most likely suffering from which illness?
Correct
Incorrect
Explanation:
Measles is a highly contagious, viral infection that is most common in children. It is characterized by fever, cough, coryza, conjunctivitis, enanthem (Koplik´s spots) on the buccal or labial mucosa, and a maculopapular rash that spreads cephalocaudally. Diagnosis is usually clinical. Treatment is supportive. Vaccination is highly effective.
-
Question 7 of 100
7. Question
1 pointsApproximately 30%-50% of infants born to Chlamydia-positive mothers will have which of the following?
Correct
Incorrect
Explanation:
Chlamydial genital infection is reported in 5%-30% of pregnant women, with vertical transmission to >50% of their infants at birth. An infant born to a mother with active chlamydial infection has a 50%-70% risk of acquiring infection at any anatomical site. Approximately 30%-50% of infants born to Chlamydia-positive mothers will develop conjunctivitis, and at least 50% these children will also have nasopharyngeal infection. Infants born to women with chlamydial infection may also develop associated pneumonia, but this affects only 10%-20% of this population.
-
Question 8 of 100
8. Question
1 pointsA 9 year old child with sickle cell disease presents with left leg pain and a high fever. He has been refusing to walk since yesterday. On physical examination, his temperature is 39.8 C (103.6F), blood pressure is 122/68 mmHg, pulse is 102 per min, and respirations are 20 per min. His left femur is tender to palpation 3 cm above the left knee, and there is marked soft tissue swelling. A plain film of his left leg is normal. A bone scan shows increased uptake around the metaphysic of the left femur. The most likely pathogen is which one of the following?
Correct
Incorrect
Explanation:
Osteomyelitis is a pyogenic infection of the bone. The pathogenesis of the disease is similar to septic arthritis, with the origin of infection occurring from hematogenous spread, direct extension of a local infection, or direct inoculation of bone either from trauma (e. g. puncture wound or open fracture) or surgical manipulation. In children, the most frequent presentation is acute hematogenous spread. The most common location of osteomyelitis is the metaphysis of the distal femur and proximal tibia. The most prevalent pathogens are the same as those seen in septic arthritis. Staphylococcus aureus is the most common pathogen, with group A beta-hemolytic streptococci a distant second. Neonates are at risk for group B beta-hemolytic streptococci.
Patients with sickle cell disease are at risk for infection by Salmonella and other gram-negative bacteria, and patients in the 18- to 48-month age range are at increased risk for acute recurrent Salmonella osteomyelitis. Salmonella osteomyelitis frequently involves multiple sites and creates punched-out destructive lesions of the metaphysis and diaphysis. However even in patients with sickle cell disease, Staphylococcus aureus is still the most common pathogen for osteomyelitis. -
Question 9 of 100
9. Question
1 pointsA 5 year old girl presents with a 2 day history of low-grade fever and headaches, followed by an onset of erythematous facial flushing, most prominent on her cheeks. Examination reveals a macular rash on her face (particularly cheeks), trunk and extremities and arthritis of wrist joints. The most likely pathogen is
Correct
Incorrect
Explanation:
Erythema infections m (fifth disease, parvovirus B19 infection) is a contagious viral infection that causes a blotchy or raised red rash with mild illness. Erythema infectiosum is caused by human parvovirus B19 and occurs most often during the spring months, often in geographically limited outbreaks among children and adolescents. Infection is spread mainly by breathing in small droplets that have been breathed out by an infected person. The infection can also be transmitted from mother to fetus during pregnancy, rarely resulting in stillbirth or severe anemia and excess fluid and swelling (edema) in the fetus (hydrops fetalis).
Symptoms begin about 4 to 14 days after infection but many children have none. However, some have a low fever and feel mildly ill for a few days. Seven to 10 days later, children develop red cheeks that often look like they have been slapped as well as a rash, especially on the (arms, legs, and trunk but not usually on the palms or soles. The rash [can be itchy and consists of raised, blotchy red areas and lacy patterns, particularly on areas of the arms not covered by clothing, because the rash may be worsened by exposure to sunlight. -
Question 10 of 100
10. Question
1 pointsA 9 month old child develops low-grade fever and sunburn-like erythema over much of the body, but most prominently in the intertriginous areas. Within 36 hours, sheet-like desquamation is noted in the flexures and around the mouth. The mucous membranes are spared. Which of the following is the most likely diagnosis?
Correct
Incorrect
Explanation:
Staphylococcal scalded skin syndrome is an acute epidermolysis caused by a staphylococcal toxin. Newborns and children are most susceptible. Symptoms are widespread bullae with epidermal sloughing.
The initial lesion is usually superficial and crusted. Within 24 h, the surrounding skin becomes painful and scarlet, changes that-quickly spread to other areas. The skin may be exquisitely tender and have a wrinkled tissue paper-like consistency. Large, flaccid blisters arise on the erythematous skin and quickly break to produce erosions. Intact blisters extend laterally with gentle pressure (Nikolsky´s sign). The epidermis may peel easily, often in large sheets. Widespread desquamation occurs within 36 to 72 h, and patients become very ill with systemic manifestations (eg, malaise, chills, fever). Desquamated areas appear scalded. Loss of the protective skin barrier can lead to sepsis and to fluid and electrolyte imbalance.
Diagnosis is by examination and sometimes biopsy. Treatment is antistaphylococcal antibiotics and local care. Prognosis is excellent with timely treatment. -
Question 11 of 100
11. Question
1 pointsKoplik´s spots are indicative of which one of the following viral infections?
Correct
Incorrect
Explanation:
Measles present classically with Koplik´s spots accompanied by conjunctivitis, photophobia, coryza, fever, and cough. The diagnosis of measles infection is primarily clinical. However, owing to the widespread administration of a live attenuated measles virus vaccine, the frequency of the disease has been reduced to virtually nil.
-
Question 12 of 100
12. Question
1 pointsA 6 year old black girl is brought to the emergency room because of intractable vomiting and lethargy. Elevated transaminase levels and hepatomegaly were noted. Patient was given aspirin for chickenpox one week prior to admission. The most likely diagnosis is
Correct
Incorrect
Explanation:
Reye´s syndrome is a biphasic condition that is common after a viral illness and is associated with aspirin ingestion, especially in children. Acute fatty infiltration of the liver associated with dysfunction of the liver and possible encephalopathy can occur. Liver biopsy shows non- inflammatory, panlobular, hepatocellular accumulation of lipid droplets. Black, inner city children are commonly affected. Kawasaki´s syndrome is a mucocutanous lymph node disorder associated with an erythematous, desquamative exanthem, conjunctivitis, and fever. It is common in children. Patients may also present initially with lethargy, among other signs. Of greatest concern in this disease is the risk for coronary artery aneurysms. Aspirin plays a therapeutic rather than etiologic role, helping to reduce the risk for arterial abnormalities. Hepatitis C is transmitted via blood transfusions and infected needles. It is an RNA virus which infects the hepatocytes and causes inflammation and necrosis of the liver. It is insidious in onset, and infected individuals may be asymptomatic with normal lab values (e.g., normal transaminase levels). It is uncommon in children. Varicella zoster causes either chickenpox, which is common in child- hood, or shingles, which is more likely to occur in elderly and immunocompromised individuals. Both conditions are characterized by cutanous, painful lesions of specific dermatomes of the body Aspirin and its derivatives should not be administered to children with chickenpox because it increases the risk for Reye´s syndrome. Cystic fibrosis is an autosomal recessive disorder which causes pulmonary symptoms and symptoms of malabsorption in children and young adults. It rarely causes severe liver dysfunction. -
Question 13 of 100
13. Question
1 pointsA boy aged 10 years develops erythema infectiosum and symmetrical polyarthralgias. What is the most likely diagnosis?
Correct
Incorrect
Explanation:
Erythema infectiosum with polyarthralgias is typical of parvovirus B19 infection in children. Juvenile rheumatoid arthritis is not associated with this skin rash. This rash is also not associated with Borrelia burdorferi (the causative agent of Lyme disease) or hepatitis B. Haemophilus influenzae is not typically associated with either polyarthralgias or erythema infectiosum.
-
Question 14 of 100
14. Question
1 pointsA girl aged 10 years has fever, rash, and “red tongue. Examination shows asymmetric cervical adenopathy, conjunctivitis, fever of 103° F, and a scarletiniform rash on the trunk. Her palms and soles are red. The most likely diagnosis is
Correct
Incorrect
Explanation:
Kawasaki´s disease is common in children. It is a mucocutaneous lymph node syndrome associated with coronary artery aneuryms, leading to myocardial infractions one to two percent of the time. The scarletiniform and morbilliform rash is common. Toxic shock syndrome is caused by staphyloooccus aureus and is associated with tampon use and post surgical infection. Scarlet fever presents in a similar way, however, as well as conjunctivitis, would be unusual. Streptococcus is the offending organism involved. Chicken pox would be a vesicular rash initially, and then become crusty. It is caused by Varicella Zoster virus, which is morphologically identical to, yet antigenically different from, other herpes viruses. A viral exanthem would be more confluent and diffuse. It would not be associated with a scarlet tongue.
-
Question 15 of 100
15. Question
1 pointsA 3-year-old girl has runny nose, sneezing, and mild fever since 2 weeks. Severe bouts of coughing followed by vomiting are present. Immunizations are deficient. The mother is a smoker. The child gasps noisily after each bout of cough. No retractions are present. Mild conjunctival erythema is present. Temperature = 38.2°C, pulse = 130/min and respirations are 32/min. Which of the following would most likely have prevented this condition?
Correct
Incorrect
Explanation:
The child presented to her general practitioner with the characteristic signs and symptoms of whooping cough: upper respiratory tract symptoms with the development of paroxysmal coughing, marked by episodes of coughing with an inspiratory and noisy “whoop” sound. Whooping cough is one of the most common vaccine- preventable disease among children. Despite the widespread use of vaccines, the disease has made a comeback in recent years. This child is at risk for whooping cough because she did not receive the DTP/DTaP vaccination and was exposed to a symptomatic adult. (Adults with a mild respiratory illness that is unrecognized as pertussis infections are frequently the source of infection. Immunization with DTP/DTaP would most likely have prevented the child´s infection with Bordetella pertussis (the causative agent of whooping cough).
Whooping cough has three stages. Stage 1 (catarrhal) begins with insidious, catarrhal upper respiratory tract symptoms (rhinitis, sneezing, irritative cough) that last 1-3 weeks. Stage 2 (paroxysmal) is marked by a paroxysmal cough characterized by 10-30 forceful coughs and ending in a loud inspiratory “whoop”. Vomiting commonly follows a paroxysm. Cyanosis, sweating, and exhaustion often accompany the coughing. This stage often lasts 2-4 weeks, with gradual improvement in stage 3 (convalescence), the patient experiences an irritative, chronic cough that may persist for several weeks or even months.
Antiviral drugs or use of bronchodilators would not have prevented the child´s infection with B. pertussis. Smoking in the home has been linked to chronic cough, asthma, and respiratory infections in children. However, smoking is not the cause of this child´s condition, and reducing smoking in the home would not have prevented whooping cough. Parents should however, still be advised to cease smoking or, at the very least, smoke outdoors and away from children.
Erythromycin for 14 days is the treatment for whooping cough if suspected or confirmed. -
Question 16 of 100
16. Question
1 pointsAn 8 year old girl was prescribed penicillin for a severe sore throat. She developed a diffuse maculopapular rash, shortness of breath, and wheezing. She is successfully treated in the ER for this condition. The girl complains dysuria and increased urinary frequency three months later. Urine Gram´s stain reveals gram negative rods. What antibiotic could be safely given to this patient?
Correct
Incorrect
Explanation:
This child was treated with penicillin for her sore throat but she developed an anaphylactic reaction. Therefore, this time she can´t be given penicillin for her urinary tract infection. Instead, she should be given aztreonam which is a monobactam. Since the basic ring structure of aztreonam is different from penicillins, there is no cross-allergenicity, and it can be safely given to those who have had severe reactions to penicillins. Aztreonam is highly active against gram negative bacteria but has no activity against gram positive bacteria or obligate anaerobes. It is β-lactamase resistant. In general, there is cross-allergenicity between all the penicillins, since most people react to breakdown products of the β-lactam ring structure common to all penicillins. Patients who have experienced an allergic reaction to penicillin may also be sensitive to cephalosporins. Ampicillin is a third generation penicillin. It is broad spectrum penicillin that is active against certain gram-negative and gram-positive bacteria. It is also one of the main antibiotics (along with clindamycin) that can lead to antibiotic-induced pseudomembranous colitis. Cefoperazone is a third generation cephalosporin. It is a broad spectrum cephalosporin that is active against many gram-negative and gram-positive bacteria. It contains a methylthiotetrazole side chain that can cause a vitamin K deficiency and disulfiram like reaction to alcohol. Many people also experience diarrhea as a side effect. It is al drug of choice in people with impaired renal function because 60% is eliminated by the biliary route. Cephalexin is a first generation cephalosporin. It is mainly active against gram-positive bacteria.
Methicillin is a second generation penicillin. It is active against many gram positive organisms. When given in high doses for more than 2 weeks it can cause interstitial nephritis, hepatitis, and neutropenia. -
Question 17 of 100
17. Question
1 pointsA 3 month old infant has fever, cough, and poor feeding for 3 days. On examination the baby appears ill and has a temperature of 38.9°C (102°F) and a respiratory rate of 32/min. CXR shows bilateral patchy infiltrates in the lungs. The most likely infectious agent has which characteristic?
Correct
Incorrect
Explanation:
Respiratory syncytial virus is the most common cause of bronchiolitis and pneumonia in children younger than 1 year of age. Outbreaks occur seasonally in winter and early spring. Infection does not result in lasting immunity, and reinfection can occur. The virus belongs to the Paramyxoviridae, a group of negative-sense, single-stranded RNA viruses that are nonsegmented, helical, and enveloped. Double stranded DNA naked, icosahedral virus describes the adenoviruses, which are capable of causing conjunctivitis, pharyngitis, upper respiratory disease, and gastroenteritis. However, these agents are not common causes of bronchiolitis and pneumonia in infants. Negative sense, single stranded RNA with eight segments, helical, enveloped virus describes the orthomyxoviruses, which include influenza virus. Influenza is a systemic illness characterized by the sudden onset of fever, headache, myalgias, malaise, and prostration, followed by cough, nasal obstruction, and sore throat. The lower respiratory tract may also be involved. Positive sense, single stranded RNA, helical, enveloped virus describes the coronaviruses, which cause the common cold (nasal obstruction and discharge, sneezing, mild or no fever, occasional sore throat, and/or cough) and acute pharyngitis (sore throat, with or without cervical adenopathy, ulceration, and conjunctivitis). SARS-Coy also belongs to this family and causes a seasonal acute respiratory distress syndrome, which is usually imported into this hemisphere from the Far East. Positive sense, single stranded RNA naked, icosahedral virus describes the picornaviruses and caliciviruses. Within the Picornaviridae, rhinoviruses cause the common cold but would not be common agents of bronchiolitis and pneumonia in infants. -
Question 18 of 100
18. Question
1 pointsA 5 year old boy is brought to the ER in extreme respiratory distress with a temperature of 39.9oC (103.8oF). He is drooling and has difficulty swallowing, and inspiratory stridor is noted on examination. The vaccine that would have protected against this condition contains which substance to stimulate isotype switching?
Correct
Incorrect
Explanation:
Epiglottitis is the most common disease of the upper respiratory tract produced by Haemophilus influenzae type B, a gram negative encapsulated rod. It is also common cause of otitis media in children and may cause bronchitis bronchiolitis, and pneumonia in adults. The incidence of serious disease caused by H. influenzae type B has decreased greatly with the introduction clan effective vaccine. The vaccine is composed of the polyribitol phosphate capsular polysaccharides coupled to a protein carrier molecule (diphtheria toxoid), given to children between 2 and 15 months of age. The purpose of conjugating the polysaccharide to protein is to stimulate TH2 cells to create the cytokines that will stimulate B cells to switch is of types of antibody. Without T-cell help, B lymphocytes can only make IgM, and IgM is not the best immunoglobulin for protection from a mucosal surface pathogen such as H. influenzae. This patient had not received the HiB conjugate vaccine and therefore was susceptible to this organism. Endotoxin is a component of the outer membranes of all gram-negative bacteria. H. influenzae possesses endotoxin, but it is not included in the vaccine because the molecule is not strongly immunogenic. Hemagglutinin molecules are used by both bacteria and viruses for adherence to host cells, however this is not present in the HiB vaccine. Mycolic acid is present in the cell wall region of acid-fast bacteria. H. influenzae does not possess mycolic acids and these molecules are not present in the HiB vaccine. Polyribitol phosphate is the capsular material of Haemophilus influenzae that is conjugated to the diphtheria toxoid. The role of the capsular polysaccharide is to serve as a hapten to which B lymphocytes will respond by producing antibody. However, in the absence of a conjugated protein, the only antibody isotype that would be made is IgM.
-
Question 19 of 100
19. Question
1 pointsA 5 year old child is brought into a clinic by his mother. The child received an MMR vaccine a week ago and now the child has an unusual rash distributed in a unilateral dermatomal pattern. The most likely explanation for this occurrence is
Correct
Incorrect
Explanation:
This rash is diagnostic of the rash of shingles, the second manifestation of infection with the varicella zoster virus. After the initial episode of chickenpox (primary infection), the varicella zoster virus develops a latent infection in the sensory ganglia cells. If the latent virus is reactivated, herpes zoster develops. The majority of cases of varicella (chickenpox) presents in children, who have a pruritic rash consisting of papules, vesicles, pustules, and crusted lesions in crops in various stages. Herpes zoster (shingles) mainly presents in the elderly population as unilateral painful vesicles restricted to one dermatome.
Exposure of a “naive” individual to chickenpox would result in a case of chickenpox, which has different symptomatology from that described. Exposure of a “naive” individual to shingles would result in a case of chickenpox, which has a different symptomatology from that described. MMR is an attenuated live virus vaccine against measles mumps and rubella. It does not have a constituent that would cause a shingles-like rash. Contamination with live virus is not a possibility because any contamination with varicella zoster virus would have resulted in a case of chickenpox, not shingles. -
Question 20 of 100
20. Question
1 pointsA 16-year-old delivers a 43 cm-long baby girl who weighs 2,000 grams. She had not received any prenatal care as she was hiding her pregnancy. Her Apgar score are 5 and 6 at 1 and 5 minutes, respectively. The child has an apical heave, as well as a thrill at the second left intercostal space. A loud systolic and diastolic continuous rasping murmur is heard at the upper left sternal border. Echocardiography reveals a hyperdynamic left ventricle and abnormal flow. CXR shows prominence of the pulmonary artery and increased pulmonary vascular markings. Which of the following maternal infections is most likely the cause of this cardiac defect?
Correct
Incorrect
Explanation:
This patient has a patent ductus arteriosus, which results from failure of closure of the ductus arteriosus postnatally. This results in blood flowing from the aorta to the pulmonary artery. Patent ductus arteriosus is twice as common in girls. It is also associated with maternal rubella infection and with prematurity. Patent ductus arteriosus can be beneficial in providing pulmonary blood flow when there is an associated right ventricular outflow tract obstruction, or in supplying systemic flow in coarctation of the aorta. A small patent ductus arteriosus presents no symptoms.
Large patent ductus arteriosus can cause heart failure similar to large ventricular septal defects. Large patent ductus arteriosus causes a wide pulse pressure and bounding arterial pulses. An apical heave may be observed, in addition to a thrill at the second left intercostal space. The characteristic murmur is described s a “machinery” or to- and-fro, murmur heard in both systole and diastole. It can be less prominent or not heard in diastole in infants or in the presence of increased pulmonary vascular resistance. Chest radiographs show prominent pulmonary artery and increased pulmonary vascular markings.
Heart size may be normal or slightly enlarged. Echocardiography demonstrates the patent ductus arteriosus with its abnormal flow and a hyperdynamic left ventricle. Patent ductus arteriosus in premature infants may close spontaneously or respond well to indomethacin. It rarely closes spontaneously alter infancy, and surgical or catheter closure is recommended. Complications include congestive heart failure and infective endocarditis.
Early intrauterine herpes simplex virus infection with disturbance of embryogenesis occurs during the first 8 weeks of gestation. It may result in abortion, cutaneous scarring, or the following severe congenital malformations: intrauterine growth retardation, severe brain damage intracranial calcification, microcephaly, hydranencephaly, microphthalmia, chorioretinitis, retinal dysplasia, and short digits. Patent ductus arteriosus is not a typical malformation in this syndrome. Human papilloma virus is not a known cause of congenital malformations in newborns. Infection with these viruses may, however occur during birth by passage through an infected birth canal, and these babies are at risk of developing laryngeal papillomatosis or cutaneous/genital warts during the first 2 years of life.
Congenital syphilis develops when a fetus becomes infected with the spirochete Treponeme pallidum sometime after the 16th week of pregnancy. It can clinically be divided into early congenital syphilis (appearing before age 2) and late congenital syphilis (appearing alter age 2). The manifestations of early congenital syphilis are generally infectious (rhinitis, maculopapular exanthem, lymphadenopathy, hepatosplenomegaly, osteochondritis) and, since there is no primary stage, may resemble those of acquired secondary syphilis, but are more widely distributed, more severe, and last longer. The lesions of late congenital syphilis are either a hypersensitivity reaction of the host or scars and deformities resulting from the infection. Congenital varicella zoster syndrome (choice E) is caused by in utero infection of the fetus by varicella zoster virus during an episode of maternal varicella. It manifests with low birth weight congenital eye defects (microphthalmia, cataracts, and chorioretinitis);
encephalomyelitis, often with severe brain damage; hypoplastic limbs with flexion contractures; cutaneous scars; micrognathia; pneumonitis; and increased susceptibility to infection. Cardiac abnormalities are not a prominent lecture of this syndrome. -
Question 21 of 100
21. Question
1 pointsA 2-year-old has 6 days old spiking fever with swelling of his hands and feet on the third day of fever, and a maculopapular rash on his torso that developed yesterday. No other significant history. A polymorphic maculopapular rash is present on his torso, with periungual desquamation. He has bilateral conjunctival injection, and there is presence of cervical lymphadenopathy. Vitals are normal, except for a temperature of 40°C (104° F). Which of the following is the most appropriate screening test to prevent mortality from this condition?
Correct
Incorrect
Explanation:
This child´s presentation meets the diagnostic criteria for Kawasaki disease. Also called mucocutaneous lymph node syndrome. This is a systemic vasculitis of unknown etiology and the most common cause of acquired heart disease in children in the United States and Japan. Kawasaki disease can cause coronary artery abnormalities, including coronary aneurysms in 20-25% of patients who are not treated and in approximately 5% of patients who are treated with intravenous immunoglobulin and aspirin. Therefore, ` echocardiography is routinely performed in all patients with Kawasaki disease to detect dilatation and aneurysm of the coronary vessels.
Children with coronary abnormalities are at high risk for myocardial infarction, sudden death, coronary thrombosis, and myocardial ischemia within the First year after onset, and also have a higher lifetime risk .in the long term.
Diagnostic criteria for Kawasaki disease include fever>5 days (often to 40°C [104°F] or higher), plus four of the following:- Bilateral nonexudative conjunctival injection
- Changes of the oral mucosa (e.g., erythematous, dry, fissured lips), injected pharynx, or strawberry/tongue
- Polymorphic exanthem (maculopapular, scarlatiniform, erythema.
- multiforme) of the trunk,
- Changes in the hands and the feet, including redness, swelling, and the presence or absence of induration.
- Periungual desquamation occurs in the subacute phase.
Bone marrow biopsy, computed tomography of the brain, and cerebrospinal fluid analysis are not routinely performed in Kawasaki disease patients and will not prevent the morbidity or mortality from coronary damage seen in this vasculitic condition.
Ultrasound of the liver can confirm the diagnosis of gallbladder hydrops, which can occur in Kawasaki disease when there is a presence of elevated bilirubin levels and right upper quadrant mass. This is usually a self-limited complication in Kawasaki disease and usually resolves within 2 weeks of standard therapy. Gallbladder hydrops is not a significant source of morbidity or mortality.
Kawasaki disease can cause coronary artery, abnormalities, including coronary aneurysms. Children with coronary abnormalities are at high risk of myocardial infarction, sudden death, coronary thrombosis, and myocardial ischemia. Therefore, echocardiography is routinely performed in all patients with Kawasaki disease to detect dilatation and aneurysm of the coronary vessels. -
Question 22 of 100
22. Question
1 pointsA 9-year-old boy just has a erythematous maculopapular rash that has started on the head and is spreading downward to his trunk. He also complains of a fever, cough, and a runny nose for the past 5 days. On physical examination, his temperature is 38.2 °C (100.7 oF), blood pressure is 88/56 mm Hg, pulse is 76/min, and respirations are 16/min. Rash is shown below. Which of the following is the most likely diagnosis?
Correct
Incorrect
Explanation:
Measles (rubeola) is a very contagious, exanthematous respiratory disease with a pathognomonic enanthem. A live attenuated measles vaccine became available in 1963 in the U.S. and elsewhere, and measles is now an unusual disease in countries where this vaccine is widely used. However, measles continues to occur sporadically in mini-epidemics.
The measles virus is transmitted by respiratory secretions, predominantly through exposure to aerosols but also through direct contact with larger droplets. Patients are contagious for 1-2 days before the onset of symptoms until, 4 days after the appearance of the rash. Measles begins with a few-day respiratory prodrome of malaise, cough, coryza, conjunctivitis, nasal discharge, and increasing fever.
Just before the onset of the rash, Koplik´s spots appear as 1-to 2-mm blue-white spots on a bright red background shown as follows:
The characteristic erythematous, non-pruritic, maculopapular rash of measles begins at the hairline and behind the ears, spreads down the trunk and limbs to include the palms and soles, and often becomes confluent. At this time, the patient is at the most severe point of the illness. By the 4th day, the rash begins to fade in the order in which it appeared. Brownish discoloration of the skin and desquamation may occur later. Fever usually resolves by the 4th or 5th day after the onset of rash; prolonged fever suggests a complication of measles. Lymphadenopathy, diarrhea, vomiting, and splenomegaly are common features. Therapy for measles is largely supportive and symptom based.
Erythema infectiosum, or Fifth´s disease, is a mild illness caused by parvovirus B-19. It usually begins as a marked erythema of the cheeks, giving a “slapped cheek´ appearance.
Hand-foot-mouth disease presents with a prodrome of fever and anorexia, followed by ulcers on the tongue and oral mucosa and a vesicular rash on the hands and feet. Roseola infantum presents with an abrupt onset of a high fever, with temperatures up to 39.5 °C-41.1 °C (103-106°F). A maculopapular rash appears on the trunk on the 3rd or 4th day when the fever breaks.
Rubella causes a mild syndrome, which is characterized by an erythematous, maculopapular, discrete rash, generalized lymphadenopathy, and fever. It can cause congenital rubella syndrome in the infant if the pregnant mother is infected with the virus. -
Question 23 of 100
23. Question
1 pointsA 7-year-old boy has sudden onset of headache, fever, vomiting, and a skin rash after returning from an outing trip. He had experienced multiple insect bites and got ticks removed from his body. A rose-colored rash with multiple erythematous macules exists on his lower legs and hands, and rare petechiae interspersed between them. His mucosae are dry. How will you treat?
Correct
Incorrect
Explanation:
Rocky Mountain spotted fever is a rickettsial disease characterized by fever, headache, and a rash. It is caused by Rickettsia rickettsii the vector a tick, and the reservoir rodents and mammals. Clinically, the onset is nonspecific. The patient may complain of fever, myalgias, nausea, and vomiting. The typical triad of the illness is headache, fever, and a rose-colored, blanching, maculopapular rash that begins on the extremities and spreads centripetally to the entire body. The erythematous macules soon turn into petechiae as the result of vasculitis. Diagnosis is established based on history and physical examination. Early in the disease, there is no single laboratory´ study to confirm the diagnosis, but in the convalescent phase, serology may be confirmatory. Thrombocytopenia, a low leukocyte count, and low serum sodium aid in making the diagnosis.
Treatment should be initiated on clinical diagnosis alone as early as possible. Tetracycline or doxycycline is the antibiotic of choice.
These may be given even to patients younger than 8 years of age, because it is a potentially fatal disease (10% mortality); tooth discoloration and growth delay are dose-dependent and it is unlikely that the patient will need multiple courses of the drug.
Amoxicillin is the treatment of choice for Borrelia burgdorferi infection (Lyme disease) in children younger than 8 years of age. In adults, doxycycline is also recommended for Lyme disease. It is not active against Rickettsia rickettsii because it is an intracellular organism that dots not have a cell wall to destroy.
Amoxicillin/clavulanate is not active against Rickettsia rickettsii for the same reason as amoxicillin alone. Adding a beta- lactamase inhibitor would not change the spectrum to cover rickettsia infections.
Ceftriaxone is the treatment of choice for the secondary manifestations of Lyme disease, such as neurologic and cardiac involvement. It is not active against Rickettsia rickettsii for the same reason as the other cell-wall synthesis inhibitor antibiotics.
In patients not able to take tetracyclines, chloramphenicol should be considered, but because of its side-effect profile, it is a second-line option. Potential adverse effects of chloramphenicol include bone marrow suppression, plastic anemia, leukemia and Gray baby syndrome.
Penicillin G is the treatment of choice for congenital and tertiary syphilis. It has no activity against rickettsial infections because of its mechanism of action. -
Question 24 of 100
24. Question
1 pointsA 6-year-old boy has 2 day history of fever, anorexia, loose stools, and yellow skin color. He attends a large daycare center. On physical examination, his temperature is 38.1 °C (100.7 °F), blood pressure is 88/56 mm Hg, pulse is 74/min, and respirations are 15/min. Initial laboratory evaluation reveals a total bilirubin of 1.8 mg/dL and alanine aminotransferase of 764 U/L. Neither of his parents, nor any of his siblings are sick. Which of the following is the most appropriate diagnostic test?
Correct
Incorrect
Explanation:
This boy most likely has hepatitis A. Although it is more common in developing countries, it is still a common infection in the developed world. It is transmitted by the fecal-oral route when contaminated food or water is congested. The incubation period is about l5-40 day. Ninety percent of children acutely infected with hepatitis A are asymptomatic. The virus is shed in the feces 2-3 weeks before the onset of jaundice until approximately l week after onset. Most infected children are infectious for a long time before they are symptomatic. Therefore, hepatitis A is very difficult to control. Large outbreaks frequently occur in daycare centers. Diagnosis is best made by determination of IgM levels against hepatitis A virus.
The presence of IgM for hepatitis A suggests acute infection; this antibody peaks at 4-6 weeks and does not persist beyond 6 months. IgM is produced in the primary infection, but for most viral infections, including hepatitis A, it persists for a life time. Therefore, the presence of IgM against hepatitis A could mean a previous infection and is not diagnostic of a current infection. Stool culture for hepatitis A cannot be used to confirm the diagnosis.
Prophylaxis with immunoglobulin is recommended for the household and close contacts of the infected person within the first 2 weeks of exposure. Strict hand washing is also very important. There is a killed-virus vaccine available that provides immunity to travelers to developing countries if given in two doses 6-12 months apart.
The presence of hepatitis B surface antigen in the serum signifies an active/current infection with hepatitis B. IgG against hepatitis B surface antigen comes from an immunologic response to either the hepatitis B virus surface antigen from an infection, or from the hepatitis B virus vaccine. -
Question 25 of 100
25. Question
1 pointsA 15-year-old boy has a temperature of 38.4 oC (101 oF) and low back, wrist and knee pain. He had a sore throat 1 month earlier. His arthritis is diffuse. Pea-sized swellings are noted over the skin on his knees. He has a serpiginous erythematous area on his anterior trunk. His blood and throat cultures are negative, and his CBC is unremarkable. His antistreptolysin-O (ASO) titer is high. Which of the following is the most appropriate therapy?
Correct
Incorrect
Explanation:
This patient has acute rheumatic fever from group A streptococci. He has migratory polyarthritis, erythema marginatum, and subcutaneous nodules. Other features absent in this patient are chorea and carditis. His ASO titer indicates recent infection with Streptococcus. It is advisable to administer penicillin for the infection. The arthritis can be managed with salicylates.
Acetaminophen may be needed to keep his temperature down and prevent a febrile seizure. However, this is not a treatment also; a nonsteroidal anti-inflammatory drug is needed for the arthritis.
Aspirin will combat the arthritis, but the infection will A remain untreated. Such a course will expose the patient to potential- rheumatic heart disease, including severe mitral stenosis, in the future.
The penicillin alone will cover the infection and eliminate the risk of rheumatic heart disease; however, the patient also has a painful arthritis that needs to be addressed.
Supportive care is not an option for this patient. Even if he is allergic to penicillin, an alternative coverage for gram-positive bacteria is mandatory. Prophylactic therapy for an associated movement disorder is not needed. -
Question 26 of 100
26. Question
1 pointsTwo brothers have a lace-like pruritic rash on the trunk and proximal extremities. They complained of a sore throat, low-grade fever not exceeding 37.9 °C (100.2 °F), and headache 7 days earlier. Now they are afebrile. No pharyngitis, hepatosplenomegaly, or lymphadenopathy exists. Which of the following is the most likely causal organism?
Correct
Incorrect
Explanation:
These children most likely have erythema infectiosum, which is caused by parvovirus B19. It is a benign, sell- limited infection characterized by mild systemic symptoms and rash. The typical rash is lace-like on the trunk and proximal extremities and there is often a facial rash that gives the patient an intensely red, “slapped cheek´ appearance. In general, the treatment is supportive care only. Intravenous immunoglobulin therapy may be considered for immunocompromised patients.
Scarlet fever, caused by group a beta-hemolytic streptococci is characterized by fever, chills, headache, sore throat and sandpaper- like rash. Patients often have a “strawberry tongue,” circumoral pallor, Pastia lines, and inflamed tonsils. Desquamation of the rash may occur. Penicillin is the drug of choice.
Human herpesvirus 6 causes exanthem subitum (roseola). It is characterized by a high fever (not exceeding 41 °C [106 °F]) that lasts for 3 to 4 days with minimal physical findings. The fever resolves on the third or fourth day, and a maculopapular rash appears on the trunks, arms, neck, and face. Occipital lymphadenopathy may occur.
Treatment includes supportive therapy with antipyretics and fluids. Measles, caused by the measles virus, is characterized by a high fever, maculopapular rash, cough, coryza, conjunctivitis, and Koplik spots on the buccal mucosa. Treatment is supportive. Mumps, caused by the mumps virus, is characterized by fever headache, muscle pain and malaise. The prodrome is usually followed by ` pain and swelling of the parotid gland. Treatment is supportive.
Rubella caused by the rubella virus, is characterized by a rash and enlargement and tenderness of the post occipital, retroauricular, and cervical lymph nodes. Patients may also have pharyngitis, a low-grade fever, and upper respiratory infection symptoms. Treatment is supportive. -
Question 27 of 100
27. Question
1 pointsA 9-month-old girl has a rash which unlike as it was a week before is now very itchy and the infant has been rubbing and scratching, especially at night. It is maculopapular excoriated rash primarily on the face but there are also patches on the elbows. The infant has been on a soy formula due to an allergic-like reaction to a standard cow´s milk formula in the first months of life. Which of the following is the MOST appropriate long-term management of this patient?
Correct
Incorrect
Explanation:
The diagnosis is atopic eczema. It is associated with elevated IgE levels and almost all affected children will develop allergic rhinitis and/or asthma. It typically begins during infancy, almost all by age 5 years. The skin is very reactive and pruritic, especially at night, and it is the scratching that leads to the characteristic skin lesions. Many factors such as foods, inhalants, and changes in humidity, infection, and irritants may exacerbate the pruritus. The initial lesions are erythematous papules with excoriation and serous exudate. Overtime, the disease becomes subacute with the presence of scaling papules. The chronic form is characterized by lichenification. The underlying problem is dry skin (xerosis), especially in winter. In infancy, the face, scalp, and extensor surfaces of the extremities are generally involved. With chronicity, the disease is localized to the flexural folds of the extremities and usually goes into remission. At this point the skin is pruritic and becomes inflamed with irritant exposure. The mainstay of treatment is appropriate skin hydration. Hydration by lukewarm, soaking baths, followed by the application of an emollient, allows for retention of moisture and provides symptomatic relief.
Antihistamines generally do not provide much benefit because histamine is only one of many mediators that can induce skin pruritus. The best use of the older antihistamines, such as diphenhydramine, is to help with sleep at night when the pruritus is usually worse.
Phototherapy is an adjunct in the treatment of atopic dermatitis. It has immunosuppressive effects and is indicated in patients with severe, widespread disease. Side effects include erythema, pain, pigmentation, and predisposition to malignancy.
The goal of therapy is to use the emollients for skin hydration and low-potency topical glucocorticoids for maintenance. A higher-potency topical steroid class may be used for short periods if there is an acute, more severe flare-up. Steroids should not be used on the face or intertriginous areas because of the formation of striae and skin atrophy.
Topical immunomodulators such as tacrolimus, inhibit the activation of a number of key cells in atopic dermatitis. The ointment is safe and effective in children and may be used safely on the face. It does not produce skin atrophy. The lesser concentration is approved for short-term and intermittent long-term use in children 2 to 15 years of age, and the higher concentration for children at least 16 years of age.
Coal-tar preparations are anti-inflammatory and antipruritic, but they are not as effective as the topical steroids or immunomodulators. Coal-tar preparations are most useful in decreasing the potency of a topical steroid being used in chronic atopic dermatitis. The common side effects are photosensitivity, folliculitis, and irritation.
mmon side effects are photosensitivity, folliculitis, and irritation. -
Question 28 of 100
28. Question
1 pointsA 10-year-old boy has a high fever, vomiting, and a sore throat along painful swallowing and malaise. Other children in his class have been sick this past week too. He has no known allergies and takes no medications. On physical examination, the child is in mild distress. His temperature is 39 °C (102.2 oF), pulse is 98/min, and respirations are 22/min. His skin is covered with yellow-white, thick exudates. He has slightly enlarged and tender cervical lymph nodes. Laboratory studies show a left shift in the differential with increased polymorphonuclear leukocytes and bands. Which of the following complications is most likely to develop despite appropriate antibiotic treatment in this patient?
Correct
Incorrect
Explanation:
This patient has streptococcal pharyngitis. The most common bacterial cause is group A beta-hemolytic streptococci- Rapid Strep antigen kits have a high degree of sensitivity and specificity. The gold standard is throat culture and it should be performed in all clinically suspicious cases. Routine blood work will reveal an elevated erythrocyte sedimentation rate, left shift of the differential white blood cell count and an increased number of polymorphonuclear leukocytes and band forms. Treatment of viral pharyngitis is symptomatic, whereas bacterial cases are treated with antibiotics. Penicillin is the antibiotic of choice and amoxicillin an acceptable alternative. Complications of streptococcal pharyngitis include rheumatic fever, glomerulonephritis, peritonsillar abscess, and retropharyngeal abscess. All but glomerulonephritis are preventable with early antibiotic treatment Pathogenesis of poststreptococcal glomerulonephritis seems to be immunologically mediated, and the incidence varies with the prevalence of nephritogenic strains in the community. The latent period between streptococcal pharyngitis and glomerulonephritis is 1 to 2 weeks.
A peritonsillar abscess should be suspected when a patient with strep pharyngitis has persistent fever and an increasingly severely sore throat. The patient drools, and the affected tonsil bulges medially. The uvula will deviate to the noninvolved site and the patient has a “hot potato” voice. Intravenous antibiotics and surgical drainage are the treatment.
Retropharyngeal abscess presents as a soft tissue mass in the lateral neck of a patient with history of recent pharyngitis. Lateral films of the neck confirm the diagnosis. Intravenous antibiotics and surgical drainage are the treatment.
Rheumatic fever is a complication of group A streptococci infection that presents as carditis and chorea in a patient with an elevated antistreptolysin C titer and a past history of untreated or inadequately treated upper respiratory tract infection.
Scarlet fever is most often the result of pyrogenic exotoxins produced by beta-hemolytic group A streptococci. Fever and sore throat precede the skin eruption by several days. The rash begins on the face and neck and spreads downward, sparing palms, soles, and the perioral area (circumoral pallor). Tiny, scaly papules appear on the trunk, giving the skin the texture of sandpaper. Fine desquamation ensues from the face and neck down whereas palms and soles often desquamate in a “gloves and socks” fashion. The erythema is accentuated in skin folds, where petechiae may also occur. The most characteristic mucosal lesion is the strawberry tongue: protruding, enlarged, g erythematous papillae and a thick white coating. A diagnosis of scarlet fever may be made clinically and is supported by throat cultures and a rising antistreptolysin O titer. -
Question 29 of 100
29. Question
1 pointsA 6-week-old has a rash involving the diaper area. The mother has 5 other children at home and says that she is very busy lending to all of them. On examination, there are erythematous, slightly scaly patches covering the buttocks and the lower abdomen. Skin creases appear spared. The baby is otherwise healthy. Physical examination reveals no lymphadenopathy, fever, or other signs of organic illness. Which of the following is the most common cause of this condition?
Correct
Incorrect
Explanation:
The clinical presentation is consistent with diaper dermatitis. The most important clinical clue is the distribution of the erythematous areas. The usual cause of diaper dermatitis is irritant contact dermatitis, resulting from the concomitant action of fecal and urinary enzymes, entrapped moisture and excessive heat in the diaper area. Avoiding tight occlusion by diapers, bathing with mild soaps and water, and frequent diaper changes constitute the essential approach to treatment. In the most severe cases, topical application of low-strength corticosteroid cream and/or zinc oxide may be indicated.
Candidiasis manifests with papules and plaques rather than extensive shiny areas of erythema. Candida infection, however, develops superimposed on diaper dermatitis. If signs of candidiasis are present a topical antifungal agent should be used in treatment.
Langerhans cell histiocytosis is a severe disease due to neoplastic proliferation of Langerhans cells. In infants, the condition manifests in its systemic life-threatening form known as Letterer-Siwe syndrome. The baby is critically ill, with fever and a diffuse scaly rash.
Psoriasis may be confused with diaper dermatitis. Psoriasis is characterized by well-demarcated silvery plaques that often involve skin folds in infants.
Seborrheic dermatitis manifests with sa1mon-colored, scaly, oily plaques that may affect the-diaper area, but most commonly involve the scalp and face. -
Question 30 of 100
30. Question
1 pointsA 9 year old girl has outward bowing of the anterior tibias, interstitial keratitis, symmetrical deafness, notched incisors, and a flattened nose. Which drug should have been used to treat the mother to prevent her child´s condition?
Correct
Incorrect
Explanation:
This question is essentially asking the reader to diagnose the condition in the mother that would lead to the development of the signs and symptoms in the child. Then the reader must determine what the treatment of choice is for the mother´s infectious condition. TORCH is an acronym referring to a group of microorganisms that lead to specific pathologic changes that occur during fetal or neonatal infection. The T stands for toxoplasma, the O for others, the R for rubella, the C for cytomegalovirus, and H for herpes simplexvirus. The “others” portion of acronym TORCH include syphilis and tuberculosis as well as a few other less common microorganisms. Pathologic manifestations of the TORCH complex include brain lesions (encephalitis and intracranial calcifications), ocular manifestations (chorioretinitis), and a variety of cardiac abnormalities. The organism Treponema pallidum causes both adult and congenital syphilis. Congenital syphilis may result in the development of typical TORCH pathologic changes as well as the following: saddle nose (flattening of the nose), Hutchinson´s teeth (notched incisors), mulberry molars (deformed molars), saber shins (outward bowing of the anterior tibias) and interstitial keratitis (vascularization of the cornea). The combination of deafness, interstitial keratitis, and notched incisors is known as Hutchinson´s triad. The treatment of choice for syphilis is consecutive daily intramuscular injections with procaine penicillin. The actual dosage and duration for the therapy are determined by the patient´s clinical presentation (chancre, secondary mucocutaneous manifestations, neurosyphilis, etc.). If no clinical staging is possible (as is usually the case), serologic staging determines the treatment regime. If the patient is allergic to penicillin, other treatment options exist, such as doxycycline. Acyclovir is used in the treatment of herpes infections.
Ciprofloxacin is used in the treatment of a variety of gram negative infections. Ganciclovir is used in the treatment of cytomegalovirus infections. Isoniazid and rifampin are the drugs of choice for the treatment of typical, drug sensitive tuberculosis. -
Question 31 of 100
31. Question
1 pointsWhich of the following is true of BCG vaccination?
Correct
Incorrect
Explanation:
BCG vaccine may be given to newborns at high risk of exposure.
The BCG vaccine is an attenuated strain; it provides approximately 70% protection.
It should not be given to these children. A low reactivity Heaf test (grade 0-1) should be documented before administration.
BCG is given at comprehensive school entry (age 11-13).
It has also found a use in stimulating the immune system for the treatment of some cancers -
Question 32 of 100
32. Question
1 pointsA child has developed a scaling yellowish rash over scalp and face. Itching is not a complain.
See picture:
What is the most likely diagnosis?
Correct
Incorrect
Explanation:
Seborrhoeic dermatitis primarily affects the scalp and intertriginous areas. It is most common in the first 6 weeks of life, but can occur in children up to 12 months of age. Involvement of the scalp is frequently termed “cradle cap”, and manifests as greasy, yellow plaques on the scalp. Other commonly affected areas include the forehead and eyebrows (as in the photo), nasolabial folds, and external ears. Involvement of skin creases, such as the nappy areas, can lead to secondary Candidal infection and maceration. The etiology is unknown. Treatment includes the use of a mild tar shampoo, oatmeal baths, and avoidance of soaps. Occasionally, a mild topical steroid may be indicated.
-
Question 33 of 100
33. Question
1 pointsA 9 year old boy presents with a cut with swelling on his arm. Swollen lymph nodes are present on physical examination. According to his mother he was most likely scratched by their neighbour´s cat. The organism is likely involved is
Correct
Incorrect
Explanation:
Cat-scratch disease is infection caused by Bartonella henselae.
Symptoms are a local papule and regional lymphadenitis. Within 3 to 10 days after a scratch, most patients develop an erythematous, crusted papule (rarely, a pustule) at the scratch site. Regional lymphadenopathy develops within 2 weeks. The nodes are initially firm and tender, later becoming fluctuant, and may drain with fistula formation. Fever, malaise, headache, and anorexia may accompany lymphadenopathy. Treatment is local heat application and analgesics. If a lymph node is fluctuant, needle aspiration usually relieves the pain. Antibiotic treatment is not clearly beneficial and generally should not be given for localized infection. -
Question 34 of 100
34. Question
1 pointsThe most frequent etiologic agent of occult bacteremia in children is
Correct
Incorrect
Explanation:
With widespread immunizaton against Haemophilus influenzae infection, Streptococcus pneumoniae has become the, predominant cause of serious bacterial infection in infants and young children. In a study of about 9000 well-appearing older infants and young children, 149 (7%) had positive blood cultures, 92% of which were S. pneumoniae.
-
Question 35 of 100
35. Question
1 pointsA 9 year old boy is brought to the pediatrician. According to her mother the left side of his mouth has started to droop over the past several days. He is also unable to close his left eye completely and complains of it burning. Review of systems reveals a cold approximately two weeks ago and recent decreased taste sensation. There is left eye ptosis and mild erythema of the left conjunctiva. His smile is asymmetrical on the left. The most closely associated infection with this condition is which of the following?
Correct
Incorrect
Explanation:
This patient has Bell palsy, a postinfectious allergic or immune demyelinating facial neuritis. Epstein-Barr virus is the preceding infection in approximately 20% of cases. 35% of patients have their symptoms resolve on their own over a period of several weeks. 10% retain mild facial weakness and 5% have permanent severe facial weakness. Therapeutic intervention should include daily and nocturnal eye lubricants to protect the cornea from drying.
Group A Streptococcus, HIV, and influenza are not associated with Bell palsy. However, herpes simplex virus, Lyme disease caused by Borrelia burgdorferi and mumps have been associated with Bell palsy. Measles is associated with subacute sclerosing panencephalitis, a chronic encephalitis of the central nervous system manifested by progressively bizarre behavior and decline in cognitive function. -
Question 36 of 100
36. Question
1 pointsA 5 year old girl has been found to have congenital hearing loss. Her mother is an 18 year old migrant farm worker who is currently at 8 weeks gestation with her second pregnancy. The mother has been found to have cervical dysplasia on her current papanicolaou (Pap) smear and has also tested positive for Chlamydia. What is the most likely cause of child´s hearing loss?
Correct
Incorrect
Explanation:
Cytomegalovirus (CMV) is the most common congenital infection and occurs in up to 2% of newborns. It is the leading cause of congenital hearing loss. The virus is transmitted by contact with infected blood, urine, or saliva, or by sexual contact. Risk factors for CMV include low socioeconomic status, birth outside North America, first pregnancy prior to age 15, a history of cervical dysplasia, and a history of sexually transmitted diseases. Infection can be primary or a reactivation of a previous infection. While the greatest risk of infection is during the third trimester, those occurring in the first trimester are the most dangerous to the fetus.
-
Question 37 of 100
37. Question
1 pointsA 2 year old girl is brought to the Emergency Room by her parents. They state that she has been suffering from a cold for five days. She later developed a barking cough and noisy breathing. Examination reveals a listless child with a mild fever, inspiratory stridor and diminished breath sounds. The most likely causative organism is which of the following?
Correct
Incorrect
Explanation:
Croup is acute inflammation of the upper and lower respiratory tracts caused most com only by parainfluenza virus type 1 infection. It is characterized by a barking cough and inspiratory stridor. Diagnosis is usually obvious clinically but can be made by anteroposterior neck x- ray. Treatment is antipyretics, hydration, nebulized racemic epinephrine, and corticosteroids. Prognosis is excellent.
-
Question 38 of 100
38. Question
1 pointsA neonate with a mother infected by the rubella virus is diagnosed with Gregg´s syndrome. Which one of the following symptoms would be the LEAST likely finding in the infant?
Correct
Incorrect
Explanation:
Gregg´s syndrome, or congenital rubella syndrome, manifests with four major abnormalities: 1) cataracts, 2) iris abnormalities, 3) glaucoma, and 4) retinopathy. Tooth abnormalities are seen in Hutchinson´s triad and are indicative of congenital syphilis.
-
Question 39 of 100
39. Question
1 pointsAt a two-month well-baby checkup, which one of the following vaccine combinations is typically administered?
Correct
Incorrect
Explanation:
At a two-month well-baby checkup, DTP (diphtheria and tetanus toxoids with pertussis), TOPV (oral attenuated poliovirus type 1, 2, and 3), and Hib (Haemophilus influenzae type b) vaccines are typically administered. DTP and Hib are administered at six months. DTP and TOPV are administered at 18 months and from four to six years. MMR and Hib are administered at the 15-month well-baby checkup. Finally MMR is administered to children at the age of 11- 12 years.
-
Question 40 of 100
40. Question
1 pointsA 9-year-old presents with a fever to 102.5o and a swollen right knee. Blood cultures and cultures of the joint fluid reveal Gram-positive cocci.
Correct
Incorrect
Explanation:
Staph aureus is by far the most common cause of joint sepsis in this age group. Group B strep is common in neonates and H. influenza in children six weeks to six years.
-
Question 41 of 100
41. Question
1 pointsWhich one of the following antimicrobial agents is most likely indicated for the prophylaxis of Haemophilus influenzae meningitis?
Correct
Incorrect
Explanation:
While ampicillin is often the drug of choice for the treatment of Haemophilus influenzae meningitis, rifampin is the drug of choice for prophylaxis. In fact, rifampin is used not only for the prophylaxis of Haemophilus influenzae meningitis, but also of Neisseria meningitides meningitis. Oxacillin is utilized for the treatment of Staphylococcus aureus meningitis. Penicillin is used for both Neisseria and Streptococcus Pneumoniae meningitis. Vancomycin is utilized in the treatment of methicillin-resistant Staphylococcus aureus meningitis. Listeria meningitis is treated with trimethoprim-sulfamethoxazole if penicillin or ampicillin fails.
-
Question 42 of 100
42. Question
1 pointsWhich one of the following organisms is most likely to cause neonatal sepsis?
Correct
Incorrect
Explanation:
Of the organisms listed, Escherichia coli is the most common cause of neonatal meningitis and sepsis. Group B Streptococci are also important causes. Haemophilus influenzae is the number one cause of epiglottitis, happening most frequently in children.
-
Question 43 of 100
43. Question
1 pointsA 2 month old infant with a fever of 104°F and lethargy is suspected to have meningitis. A lumbar puncture is performed. Elevated WBC levels are present in CSF. Cultures are pending. Organism that should NOT be suspected in this patient is
Correct
Incorrect
Explanation:
Pseudomonas aeruginosa is a Gram negative bacterium which is not common in neonatal meningitis. It can occur in debilitated patients and it carries a very high mortality. S. pneumoniae is more common in meningitis in adults (80% of cases) than in infants. H. influenzae is responsible for most cases of meningitis in children older than one month. Although vaccines might not be very effective in infants less than two months of age, it formerly caused 50% of the cases of bacterial meningitis in the U.S., but the wide- spread use of H. influenzae type B conjugate vaccine has dramatically reduced the number of cases. It is still a cause of mortality in infants. N. meningitidis is Gram negative diplococci, a cause of meningitis, and occurs most often in the first year of life. It is also a common cause of meningitis in closed populations, such as boarding schools and military barracks, especially among adolescents and young adults. Listeria monocytogenes strikes mainly immunocomprised patients, i.e., patients with renal failure, on steroid therapy, or with AIDS. This may include some neonates.
-
Question 44 of 100
44. Question
1 pointsA 6 weeks old infant is brought to the ER with a 10 day history of coughing and choking spells. WBC count is elevated with 80% lymphocytes. He is gasping for breath, is experiencing paroxysms of coughing, and vomits twice in the examination room. Encapsulated gram negative rods grow out on special media. What additional finding is most likely be present?
Correct
Incorrect
Explanation:
This child has whooping cough, or, more accurately if one goes by the chronology of the symptoms, “coughing whoop” in as much as the patient is subject to a series of coughing episodes that are followed by a rapid inspiration of air. The “whoop” is caused by the rapid passage of air through a narrowed airway. The infection of the respiratory tract by Bordetella pertussis causes a hyperreactivity of the respiratory apparatus; even the slightest stimulus will trigger a coughing episode. The offending pathogen excretes adenylate cyclase and also produces an exotoxin that inactivates the inhibitory subunit of the G protein complex, thus activating adenylate cyclase within the cells of the respiratory system. The pertussis toxin also causes lymphocytosis, activates the islets of Langerhans to cause hypoglycemia, blocks the function of immune effector cells, and increases histamine sensitivity. Bordet Gengou agar is the blood enriched (15%, versus 5% for normal blood agar) medium used for the laboratory isolation of this pathogen. Fluorescent antibody testing is used to verity the diagnosis. Cold agglutinins are produced in about 65% of patients with Mycoplasma pneumoniae. They are IgM isohemagglutinins that agglutinate autologous erythrocytes at 4°C (39.2°F). Mycoplasma pneumoniae is the most common atypical pneumonia of young adults, but would not be expected in an infant age group, nor would it present with this severity of symptoms. Erythematous rash is a common finding with streptococcal pharyngitis but Streptococcus would not cause these paroxysms of coughing. Fibrinous pseudomembranes would be found on the uvula and upper airways in a child with diphtheria, and diphtheria may cause severe episodes of coughing, but diphtheria would not cause the lymphocytosis mentioned here. In that case it would also be likely that the vignette would mention bull neck, cardiac arrhythmias, and neuropathies. Neutrophilia is not observed in whooping cough, in which lymphocytosis predominates rather than neutrophilia. -
Question 45 of 100
45. Question
1 pointsA 7 year old girl has fever, conjunctivitis, photophobia, and a cough. Pediatrician notes white spots on a bright red background on the girl´s buccal mucosa. Later a rash begins around the hairline and then spreads to the trunk and extremities. A week later the girl suddenly begins to convulse and loses consciousness. When taken to the ER involuntary movements and papillary abnormalities are noted. The most likely finding on CNS biopsy would be
Correct
Incorrect
Explanation:
The initial history given is classic for measles, with the appearance of Koplik´s spots (white spots on the buccal mucosa) followed by a rash beginning along the neck and hairline and spreading to the trunk and extremities. The sequela this child is experiencing is post infectious encephalomyelitis, which can follow either infection with measles varicella, rubella, mumps, or influenza, or vaccination with vaccinia vaccine or rabies vaccine derived from nervous tissue. Treatment is supportive, with a mortality of 15 to 40%; survivors; frequently have significant permanent neurologic deficits. The pathologic finding is perivenous microglial involvement with demyelination. Option A describes the findings of progressive multifocal leukoencephalopathy, a demyelinating disease caused by infection with JC virus especially in immunocompromised individuals. Option C describes the findings of poliomyelitis, a paralytic disease affecting the ventral horn of the spinal cord and motor cortex caused by an enterovirus (poliovirus). Option D describes the findings in herpes encephalitis which typically affects the inferomedial temporal lobes and orbitofrontal gyri. Option E describes the findings in tuberculous meningitis caused by M. tuberculosis. -
Question 46 of 100
46. Question
1 pointsA girl aged 8 years presents with complaints of crampy abdominal pain nausea, and mild diarrhea for approximately 2 weeks. The day before this visit, she vomited up a cylindrical white worm 30cm in length, which the mother preserved in a jar. The most likely means by which this organism was acquired is
Correct
Incorrect
Explanation:
This is a case of Ascaris lumbricoides infection, the only helminth infection caused by a cylindrical white worm of this remarkably large size. It is acquired directly, with no intermediate host, from fecal contamination from other human beings, and the stage of the life cycle that is transmitted is the egg. Autoinfection is a rare occurrence in helminthic infections, possible only with Strongyloides stercoralis, the threadworm, and Taenia solium, the pork tapeworm. Ingestion of cysts in contaminated water is the normal mechanism of infection used by intestinal protozoa. Ingestion of eggs from an infected pet occurs in visceral larva migrans, when the ascarids of dogs and cats infect the human, an aberrant host. It would never result in the introduction of an adult worm into the vomitus, as described here. Mosquito transmission of larvae is a means of transmission of the filarial nematodes. -
Question 47 of 100
47. Question
1 pointsAn infant aged 9 months is brought to an African Health Department to receive the second dose of OPV 2 weeks after the first vaccination. He has mild diarrhea, so the decision is made to delay further immunizations. Stool culture is unremarkable; however, a small, single stranded, positive sense RNA virus is isolated from the specimen that was not inactivated by ether. This same agent was isolated from sewage effluent the preceding week. The most likely explanation for this occurrence is which one of the following?
Correct
Incorrect
Explanation:
Poliovirus, which is a single stranded positive sense RNA virus is naked (i.e., non-enveloped) and hence will not be inactivated by lipid solvents such as ether. The live virus vaccine had colonized the intestinal tract of the infant and was still being shed 2 weeks after the earlier oral dose. This same virus, the vaccine strain, is likely to be found in sewage, as all vaccinated infants will shed virus for a period of time after immunization with oral polio vaccine (OPV).
Infection with adenovirus is not likely. Although several serotypes may cause diarrheal disease and are non-enveloped, they have a DNA genome.
Infection with hepadnavirus is not likely, although hepatitis B may cause diarrhea occasionally. It is a double stranded DNA virus. Infection with polio is not likely because mutations of the vaccine strain of the Sabin vaccine are very rare. It is possible, but not the most likely explanation for the present findings. Infection with rotavirus is not likely, although this is the major cause of diarrheal disease in infants under the age of 2 years. It is a member of the reovirus family and, as such, is double stranded. This virus causes hospitalization of 30 to 40% of the infected infants, and kills hundreds of thousands of infants in developing nations where access to hospitals is not readily available. Therapy for the watery diarrhea produced by this agent is fluid and electrolyte replacement.
-
Question 48 of 100
48. Question
1 pointsA 12 year old girl presents with right flank pain and fever. Urinalysis reveals a pH of 8, and an intravenous pyelogram shows a multiple renal calculi. She develops leg cramp, myalgias, and arthralgias after one week of therapy. What medication was most likely prescribed to this patient?
Correct
Incorrect
Explanation:
This patient most likely has a complicated urinary tract infection caused by Proteus mirabilis or Ureaplasma urealyticum. Lomefloxacin is a fluoroquinolone antibiotic indicated for the treatment of a wide variety of infections caused by both gram- negative and gram-positive organisms. This agent is primarily used in the treatment of lower respiratory tract and urinary tract infections. Lomefloxacin and other fluoroquinolone antibiotics have been associated with the development of arthropathy, myalgias, and leg cramps when administered to children younger than 18 years of age. Azithromycin is a macrolide antibiotic that is generally well tolerated. The most common side effects of this agent are mild nausea and abdominal pain. Metronidazole is an antiprotozoal and antibacterial associated with the development of seizures and peripheral neuropathy. Furthermore, this agent can produce a disulfiram-like reaction when taken with alcohol. Rifampin is an agent used to treat all forms of tuberculosis. It is also used in asymptomatic carriers of Neisseria meningitidis. Rifampin is associated with the development of hepatotoxicity, hyperbilirubinemia, porphyria, and possibly cancer.
Tetracycline is an antibiotic known to cause photosensitivity, pseudotumor cerebri, and a variety of maculopapular rashes. -
Question 49 of 100
49. Question
1 pointsA 5-year-old boy has a 4-cm-diameter, round area of alopecia in the right parietal-occipital scalp with the hairs in this area appearing to be broken off right at the hair follicle. Itching is also present. Small preauricular and suboccipital lymph nodes are palpable on the right side. KOH preparation shows chains of spores within the hair shaft. What is the treatment?
Correct
Incorrect
Explanation:
The correct diagnosis is tinea capitis. It is most commonly caused by Trichophyton tonsurans occasionally by Microsporum canis, and much less often by other Trichophyton and Microsporum species. It is common among black and Hispanic school-aged children. T. tonsurans typically creates a pattern called “black-dot´ ringworm, which is described in this case. Initially, there may be several small patches of alopecia, where the hairs are broken off close to the follicle. Regional adenopathy is common. Other presentations include diffuse scaling with little hair loss; chronic and diffuse alopecia; and an elevated, boggy, granulomatous mass known as a kerion.
Microscopic examination of a KOH preparation of hair from a border of a lesion shows tiny spores surrounding the hair shaft in Microsporum infections, and chains of spores within the hair shaft in Trichophyton infections. Definitive diagnosis may be made by planting broken-off infected hairs on Sabouraud medium. The treatment of choice is oral administration of griseofulvin for 8 to 12 weeks. It is discontinued only after repeat fungal cultures are negative.
Oral itraconazole or oral terbinafine is useful when there is griseofulvin resistance, intolerance, or an allergic reaction, but neither is FDA-approved for use in children.
Topical therapy alone is not effective but may be an adjunct to decrease the shedding of spores. For example, vigorous shampooing with a 2.5% selenium sulfide preparation may be helpful in both the patient and in potential carriers.
Topical clotrimazole would be used not for tinea capitis but for tinea corporis.
Tolnafate creams or Imidazole are used in Tinea Cruris (groin infections in adolescent males). -
Question 50 of 100
50. Question
1 pointsA 7 year old girl develops behavioral changes, and her performance starts to decline in school. Several months later she develops a seizure disorder, ataxia, and focal neurologic symptoms. She is eventually quadriparetic, spastic, and unresponsive. Death ensues within a year. At the age of 1 year this child may have had which of the following disease?
Correct
Incorrect
Explanation:
This child has subacute sclerosing panencephalitis (SSPE), which is fortunately a very rare, late complication of measles infection at an early age. The existence of this complication is part of the rationale for immunizing children against measles at an early age. It is thought that very young children´s immune and neurologic systems may permit the virus to become established in the brain. The exact mechanism of injury is poorly understood, but the brain shows encephalitis involving both gray and white matter. Forty percent of cases die within 1 year and it is thought that the disease is probably always eventually fatal. Chicken pocks causes an asynchronous rash in children, and reactivational disease in the elderly causes shingles. It is not known to be associated with central nervous system pathology. Rubella, or German measles, causes a mild 3 day rash in adults but is known to cause teratogenesis in the fetus if a mother is infected during gestation. The syndrome in the neonate (congenital rubeiia syndrome) includes patent ductus arteriosus, pulmonary stenosis, cataracts, microcepnaiy, and deafness. Mumps causes parotitis and fever in young children, and the complications of pancreatitis and orchitis when older individuals acquire the infection. It is not known to cause central nervous system pathology. Parvovirus B19 causes erythema infectiosum, or slapped cheek fever, in school-age children, and hydrops fetalis in fetuses infected in utero. It is also associated with aplastic crises in sickle-cell anemia patients, but is not known to cause central nervous system pathology. -
Question 51 of 100
51. Question
1 pointsAn 8-year-old was treated for presumptive Mycoplasma pneumoniae with erythromycin about 10 days ago. He later had a burning feeling in his mouth along swollen lips and the inside of his mouth was red. The rash consists of individual erythematous nontender macules on his face, trunk, and arms, some of which have central vesicles and bullae with areas of ulceration inside the mouth. In addition to bilateral conjunctivitis, a slit-lamp examination shows evidence of anterior uveitis. Which of the following is the MOST likely diagnosis?
Correct
Incorrect
Explanation:
The diseases listed in this question are all vesiculobullous. Erythema multiforme (EM), Stevens-Johnson syndrome, and toxic epidermolysis all represent a spectrum of disorders. Stevens- Johnson syndrome is the correct diagnosis in the question. The lesions look like those of EM but involve the face, trunk, and extremities; they are more widespread; and there must be involvement of at least two mucosal surfaces (eyes, mouth, upper airway, gastrointestinal tract, or anogenital area). Skin tenderness is generally minimal, but pain from mucosal ulceration is severe. New lesions occur in crops over 4 to 6 weeks. The bullae and erosions may result in blood loss, fluid loss, and bacterial superinfection. Eye involvement may include anterior uveitis, corneal ulceration, and panophthalmitis. Ocular scarring and visual impairment may remain. Other complications would depend on which mucosal surfaces were involved. The most common infectious cause of Stevens-Johnson syndrome is Mycoplasma pneumoniae. There are many other causes, including herpes simplex, M. tuberculosis hepatitis B, Ebstein- Barr virus, enterovirus, and other infections. In addition, other noninfectious causes include leukemia and lymphoma; the antibiotics penicillin, sulfonamides, isoniazid, tetracyclines, cephalosporins, and quinolones; the anticonvulsants phenytoin, phenobarbital, carbamazepine, valproic acid, and lamotrigine; and other causes such as radiation, captopril, nonsteroidal anti-inflammatory drugs, sunlight, pregnancy, and allopurinol.
Erythema multiforme can present as erythematous macules, papules, vesicles, or bullae to areas of confluent erythema. It is most common between ages 10 and 30 years. It tends to burn and itch. The diagnosis is clinical: doughnut-shaped, target like papules with an erythematous outer border, an inner pale ring, and a purple necrotic center. Lesions usually appear abruptly and symmetrically on the extensor surfaces of the upper extremities. These lesions commonly expand outwardly. There may be oral lesions, but no other mucous membrane surfaces are involved and it does not develop into Stevens- Johnson syndrome. The lesions usually resolve within 2 weeks. Most cases are related to- herpes simplex infection. Treatment is supportive. Toxic epidermal necrolysis (TEN) is the most severe disease in this spectrum, involving extensive toxicity and necrolysis of the mucous membranes with more than 30% of the body surface area. It is thought to represent a hypersensitivity reaction involving the basal cell epidermal layer, and is triggered by the same factors as those for Stevens-Johnson syndrome. There is widespread blister formation with erythema and tenderness, absence of target lesions, sudden onset and spread within 48 hours, and full-thickness epidermal necrosis. As may be seen with staphylococcal scalded skin syndrome (SSS), the Nikolsky sign is present-denudation of the skin with gentle tangential pressure with TEN the skin denudes only in areas of erythema, whereas with SSS it will occur on any part of the skin. In addition, the mucous membrane involvement is not as severe as with EM.
Bullous pemphigoid is rare in children. In this illness, blisters (fluid-filled, of various sizes) arise in crops, mostly on the flexural aspects of the extremities, axillae, groin, and central abdomen. The base of the blister may be normal skin or may be erythematous, eczematous, or urticarial. Infants tend to present with facial involvement and involvement of the palms and soles. Pruritus and burning may be present but constitutional symptoms are not prominent Oral lesions are less severe than the disorders presented above. This is an autoantibody disorder, and IgG and C3 may be seen at the basement membrane by immunofluorescence. There is also an inflammatory infiltrate consisting mostly of eosinophils. Primary treatment is with topical corticosteroids.
Epidermolysis bullosa represents a wide variety of hereditary blistering disorders. Differences are seen histologically from the superficial skin to deep dermal areas. All are characterized by trauma-induced blistering and by blistering in warm weather. The simplex form is autosomal-dominant and is usually present from birth.
Bullae are seen most commonly on the hands, feet, elbows, knees, legs, and scalp. Lesions heal with the scarring. The other forms may be progressive and more extensive with mucosal involvement, and may lead to squamous cell carcinoma. -
Question 52 of 100
52. Question
1 pointsAfter a weekend family camping trip in North Carolina, parents notice that their 6-year-old son has two engorged ticks on his legs present from less than 24 hours. None of the other family members have found any ticks on their bodies. The boy is in no acute distress and has no complaints. There are two moderately engorged ticks on the lower legs, with no signs of inflammation surrounding the attachment sites. The ticks are carefully removed and the physician advices the family to monitor the child for development of any signs of inflammation over the next few weeks. After drawing blood for antibody titer determination the physician explains that prophylactic antibiotic treatment is not necessary. Which of the following is most likely to develop at the site of the tick bites as a result of infection transmitted by the tick?
Correct
Incorrect
Explanation:
Erythema migrans develops 3 to 32 days after the bite of a deer tick (Ixodes scapularis) infected by Borrelia burgdorferi. The rash begins as a papule that expands to form an erythematous annular lesion with central clearing. It may be accompanied by malaise, lethargy, fever, and arthralgias.
Acrodermatitis chronica atrophicans is a late complication of Borrelia azelaic infection, seen mostly in Europe, in which the acral integument becomes atrophic and translucent. It may develop decades after the initial infection occurred.
Erythema ab igne is a reticulated brown network that develops on skin chronically exposed to high temperatures, such as i repeated applications of heating bottle to the same skin site or sitting close to radiator with heat directed toward the skin from a very short distance. It is permanent and may undergo malignant transformation to squamous cell carcinoma many years later.
Erythema annulare centrifugum is an inflammatory skin disease of unknown etiology that usually develops in adults in the form of erythematous annular plaques with central clearing and peripheral “trailing” scale. Some believe it is a reactive pattern seen as an id reaction to remote infectious foci, such as inflammatory tinea of the feet.
Erythema infectiosum, also called fifth disease, is a viral infection caused by human parvovirus B19. It is mildly contagious and affects children 3 to 12 years of age. First an erythematous malar rash appears, followed by an erythematous maculopapular eruption on the extensor extremities the next day. When the rash fades, a few days later, areas of central clearing leave behind a reticulated, lacy patterned rash that waxes and wanes for an average of 9 to 11 days.
Erythema Marginatum occurs in rheumatic fever. -
Question 53 of 100
53. Question
1 pointsA 6-year-old has a rash over the past several days without any complaints of itching. There are three erythematous, serpiginous macular lesions with pale centers on the boy´s lower extremities. In order to ultimately confirm this diagnosis, which of the following must also be present in this child?
Correct
Incorrect
Explanation:
The Jones Criteria for the diagnosis of rheumatic fever was revised in 1992 by the American Heart Association. The criteria are for the initial attack of rheumatic fever only, not for recurrences. There are five major criteria: carditis, migratory polyarthritis, erythema marginatum, subcutaneous nodules, and chorea.
The presence of any two with microbiologic or serologic evidence of a recent group A streptococcal infection is diagnostic. However, there are three circumstances in which the diagnosis may be made without strictly employing the Jones criteria: chorea as the only manifestation, indolent carditis, and recurrence of rheumatic fever.
There are two clinical minor criteria: arthralgia and fever. There are also two laboratory criteria elevated acute phase reactants and prolonged PR interval on electrocardiogram. The presence of one major and two minor criteria is diagnostic, but again, it needs to be supported by evidence of a recent group A streptococcal infection.
The patient here presents with erythema marginatum. It is a characteristic rash of rheumatic fever, but is rare. It consists of erythematous, serpiginous, macular lesions with pale centers that are not pruritic. The lesions occur primarily on the trunk and extremities but not the face. This is the only manifestation that is mentioned in the question, so we would need one more major or two minor criteria.
Thus, there is no single answer choice that would fulfill the criteria.
But that is not what the question asks. Ultimately, there has to be evidence of a recent streptococcal infection. This is an absolute requirement. This is what the question is asking.
Of the answer choices, the only one that will confirm a recent infection is the elevated anti-DNase B titer. Acute rheumatic fever typically follows a group A streptococcal infection by 2 to 4 weeks.
Many patients have no history of an antecedent pharyngitis; therefore, y evidence must be based on increasing serum anti streptococcal antibody titers. There are three different antibodies that are measured clinically: antistreptolysin O anti-DNase B, and antihyaluronidase.
Anti-DNase B is the most sensitive and specific, but 100% accuracy is approached if all three are measured. So when the diagnosis is suspected, multiple tests may need to be performed.
Arthralgia in itself does not need to present to ultimately confirm the diagnosis. The argument is the same for elevated erythrocyte sedimentation rate, fever and prolonged PR interval. -
Question 54 of 100
54. Question
1 pointsA 16-year-old boy has fever, malaise, sore throat and rash of 10 day duration. He initially had pain on swallowing. He broke out in a rash several days later. He has no significant past medical history and is otherwise in very good health. On physical examination, this very fit and athlectic young man has a temperature of 38.7 °C(101.6 °F). His tonsils are edematous and erythematous with clear exudate. The submandibular lymph nodes are enlarged. A morbilliform exanthem is present on the trunk and extremities. A rapid strep test done in the office is negative. What is the most likely cause of this patient´s disease?
Correct
Incorrect
Explanation:
This boy has infectious mononucleosis, which is characterized by fever, adenopathy, splenomegaly, lymphocytosis with atypical lymphocytes, and evidence of acute infection by Epstein-Barr virus. Because this sore throat is confused with streptococcal sore throat, ampicillin is often initiated. This triggers the appearance of an immune complex-related rash. Treatment is symptomatic. Contact sports should be avoided for 2 to 3 weeks, or until splenomegaly has resolved.
Beta-hemolytic Group A streptococci is a common cause of acute purulent pharyngitis in children. Acute purulent pharyngitis presents with fever, chills, and a sore throat. The tonsils are erythematous, edematous, and covered with purulent exudates. A quick diagnosis may be made in the office by doing a rapid strep test supported by bacterial cultures if the test is negative. On the other hand, a positive strep test is enough to start antibiotic therapy.
Penicillin is the drug of choice.
Herpes simplex virus is the cause of oro labial and genital herpes. The initial episode is a painful vesicular eruption of the oropharynx or genitalia that resolves without treatment in l to 2 weeks. The virus remains latent in cranial or spinal ganglions and, in approximately 10% of patients, will continue to recur with certain triggering events. Herpes simplex infection does not cause hepatosplenomegaly or generalized) lymphadenopathy.
Human immunodeficiency virus infection is the cause of AIDS. Patients may present with lymphadenopathy, hepatosplenomegaly, parotitis, and recurrent or persistent upper respiratory tract infections. Later, candidiasis, chronic diarrhea, herpes zoster, and other opportunistic infections ensue as the lymphocyte counts drop.
Rubivirus is the cause of rubella (German measles), a viral infection characterized by a maculopapular exanthem and tender lymphadenopathy of the post occipital, retroauricular, and cervical lymph nodes. Forchheimer spots are rose spots seen on the soft palate that may precede the skin rash. -
Question 55 of 100
55. Question
1 pointsA 7-week-old and a 3-year-old son have an itchy rash for 2 weeks. The mother also has some itchy bumps on the hands and elbows over the past several days. Calamine lotion did not benefit it. The 7-week-old has pustules on the palms and soles, and scaly, erythematous patches on the face and scalp. The 3-year-old has 1-to 2-mm scaly, linear lesions on the finger webs and on the wrists, in addition to multiple excoriated and crusted erythematous papules on the elbows, abdomen, and buttocks. Mother has similar linear lesions on the finger webs and wrists with excoriated, crusted red papules on the elbows and areole of the nipples and around the umbilicus. Which of the following is the treatment of choice for this family?
Correct
Incorrect
Explanation:
This family has scabies, an infestation caused by the mite Sarcoptes .scabiei, and the treatment of choice for the entire family is permethrin cream application overnight repeated once in 7 days. All family members and caretakers of the child should be treated. Bed linens and clothes should be laundered in hot water, or dry cleaned if laundering is not feasible.
Ivermectin) in a single oral dose is used to treat scabies in immunocompromised individuals who have overwhelming infestation that would not respond to standard topical therapy. It is not recommended for use in ordinary scabies.
Lindane cream should not be used in small infants because of its potential for neurotoxicity. It is relatively safe for use in adults, but permethrin cream has an established safety record that A makes it the treatment of choice today.
Scabies does not resolve spontaneously except in rare instances where the immune response is so strong that it eliminates the mite. Even if that were to happen, chances are that reinfestation would occur from other family members or caretakers, so treatment is necessary. It is very important to treat all family members and caretakers simultaneously for the same reason. Therefore, it is incorrect to say that no treatment is necessary because the disease will resolve spontaneously.
Precipitated sulfur in petrolatum was often used to treat scabies in infants and pregnant women but has become rather, obsolete; permethrin cream has proven to be just as safe and much easier to use.
Precipitated sulfur in petrolatum has an offensive “rotten-egg” odor that confines the treated person to the home for the duration of the treatment. In addition, it is applied daily for 7 days without bathing in the meantime. Permethrin cream is applied once to the entire skin, washed off 8 hours later, and reapplied in 7 days. It is by far more convenient and comfortable for the patient to use. -
Question 56 of 100
56. Question
1 pointsA 10-year-old boy has a long history of recurrent infection. There have been pneumonia, suppurative lymphadenitis, persistent rhinitis, dermatitis diarrhea, and perianal abscesses. Involved organisms have included Staphylococcus aureus, Serratia, Escherichia coli, and Pseudomonas. Biopsy of skin and lymph nodes have demonstrated granulomatous lesions, even though the only species isolated were those noted above. Immunoglobulin levels are higher than normal. Which of the following findings would be most helpful in establishing the diagnosis?
Correct
Incorrect
Explanation:
This child´s condition is chronic granulomatous disease, a usually X-linked recessive condition characterized by inadequate production of hydrogen peroxide, superoxide, and other activated oxygen species in neutrophils. The nitroblue tetrazolium dye reduction test demonstrates the failure of the defective neutrophils to produce these products. The clinical result of the enzymatic defect is that the neutrophils can phagocytize but not kill bacteria. Affected individuals are plagued with multiple, poorly healing infections to which their bodies respond with granuloma formation to remove organisms that would normally be controlled by neutrophils. These patients are usually treated with intermittent or continuous antibiotics; bone marrow transplant and interferon therapy have also been used.
Absent B cells and normal numbers of T cells suggests X- linked agammaglobulinemia.
High serum IgM and very low serum IgG suggests hyper IgM immunodeficiency.
Monoclonal anti-CD11 is used to diagnose leukocyte adhesion deficiency, in which the CD11 antigen on the surface of white blood cells is missing. This disorder causes a severe form of immunodeficiency that usually results in death by age 5.
Serum calcium levels can be markedly decreased in DiGeorge syndrome. -
Question 57 of 100
57. Question
1 pointsA 6-year-old child, attempting to pet a neighbor´s domestic dog while the dog is eating, is bitten in the hand. The dog has been vaccinated regularly. Which of the following steps are needed for rabies prophylaxis in this case?
Correct
Incorrect
Explanation:
Messing with a dog while the dog is eating, and being bitten in response, is considered to be a provoked attack, and thus not indicative of aggressive behavior on the part of the dog. The bite is in the hand; therefore, there is plenty of time for the virus to travel in the direction of the brain, so that therapeutic measures would still be effective should the future behavior of the dog suggest that he is rabid.
Passive immunization, active immunization, or a combination of both is not required in this low-risk situation. Had the bite been in the face, an area where rabies is prevalent, a more aggressive approach would have been justified. Killing the animal and examining the brain is the norm for wild animals that are captured alive, since their behavior cannot be deemed to be “normal” or “abnormal. It is also the appropriate step if observation of a domestic animal by a veterinarian suggests that signs of rabies are developing in the animal. -
Question 58 of 100
58. Question
1 pointsA baby is born at 34 weeks gestation. The amniotic fluid is brown and murky. The baby has an Apgar score of 5 at 5 minutes, with a temperature of 95°F (35°C). Lethargy, grunting. Apnea and bradycardia. Small, pale nodules and areas of erythematous rash are scattered all over the body. The mother had a flulike illness 1 month before delivery and she had multiple sexual partners and drug abuse. Cervix gram stain from mother shows abundant pleomorphic gram-positive bacilli. Histopathology from skin rash and nodules of neonate reveals granuloma with central necrosis. Which of the following is the most likely diagnosis?
Correct
Incorrect
Explanation:
This neonate is most likely suffering from listeriosis, caused by Listeria monocytogenes. It can be acquired by mothers, who are exposed to unpasteurized dairy products or raw vegetables, or those exposed to cattle or sheep manure, especially mothers who live on farms. Mothers who acquire Listeria may experience flulike illnesses, with headache, malaise, fever, nausea, vomiting, and generalized body aches. The bacteria have a predilection for causing amnionitis, which may then produce abortion, stillbirth, or neonatal sepsis. The presence of brown, murk, amniotic fluid may be a helpful diagnostic clue. Neonates present with a full-blown picture of neonatal sepsis, as indicated by respiratory distress (tachypnea, grunting, apnea, and retractions), temperature instability, poor feeding, and lethargy/irritability. Sometimes with severe infection, a characteristic disseminated disease in the fetus called granulomatosis infanti septicum (pathognomonic for listeriosis) can cause granuloma formation and tissue destruction in many tissues, including skin (papular or ulcerative nodules), liver, adrenal glands, lymphatic tissue, lungs, and brain. Early-onset neonatal listeriosis presents with sepsis with or without granulomatosis infantisepticum, whereas late-onset presents 2-3 weeks after birth with meningitis and sepsis.
However, blood and CSF culture confirms the diagnosis.
The microorganisms most commonly associated with early-onset sepsis include group B Streptococcus (GBS), Escherichia coli Haemophilus influenzae, and Listeria monocytogenes.
Congenital cytomegalovirus infection can cause asymptomatic (most commonly) to mild to severe disease in neonates. Some finding that clue you in to the diagnosis include small size for gestational age, hepatosplenomegaly, petechiae and purpura of the skin, jaundice, microcephaly, and seizures. Laboratory abnormalities include abnormal blood counts (especially thrombocytopenia), hemolytic anemia, elevated transaminases, and elevated serum bilirubin. CT scan of brain reveals periventricular calcifications. Other significant features include progressive sensorineural hearing loss (most com on manifestation) and chorioretinitis.
Congenital rubella syndrome (CRS) can cause multiple severe problems in infants; look for a reference to the mother having had a rash during pregnancy. The most common congenital anomalies include sensorineural deafness (60 to 75%) cataracts (white reflex), cardiac malformations such as patent ductus arteriosus or pulmonary artery stenosis, and neurologic sequelae such as meningoencephalitis, behavior disorders, and mental retardation. Infants born with CRS may manifest with intrauterine growth retardation, microcephaly, hepatosplenomegaly, thrombocytopenia, and characteristic purpura (the “blueberry-muffin” spots).
The majority of live-born neonates with congenital syphilis are asymptomatic at birth. Congenital syphilis can also cause devastating disease in newborns; look for references to rash involving palms and soles, Hutchinson molars, or bone deformities, including saber shin. Clinical manifestations after birth can be divided into early and late. Early manifestations include cutaneous lesions over the palms and soles, hepatosplenomegaly, jaundice, anemia, and occasionally snuffle (profuse watery nasal discharge) and bony abnormalities. Late manifestations include frontal Bossing, short maxilla, high palatal arch, Hutchinson triad (Hutchinson teeth [blunted upper incisors], interstitial keratitis, and eighth-nerve deafness), saddle nose, and perioral Fissures.
Even though GBS infection is the most common cause of neonatal sepsis and it presents with a sepsis picture similar to that described above, it would not present with granulomatosis infanti septicum (which is characteristic of neonatal listeriosis).
Moreover, Gram stain of the maternal cervical smear would reveal gram- positive cocci (streptococci) and not bacilli.
Neonatal herpes simplex infection can be a devastating- infection of the neonate; look for skin, eye, and mouth vesicles and often prominent neurologic involvement; Even though tuberculous nodules in tuberculosis (TB) would give a similar histologic picture of granuloma with central necrosis, nothing else in this question leads toward the diagnosis of maternal TB. Moreover, TB is transmitted neither transplacentally (like CMV, toxoplasmosis, herpes simplex, syphilis, and rubella) nor directly in- utero (like GBS and Listeria) from mother to fetus. Thus, it does not infect neonates. -
Question 59 of 100
59. Question
1 pointsA 10-month-old boy has a fever for past 3 days. The fever ranges from 38.3°C (101 to 104°F). The child has been irritable and has had two episodes of loose stools, but he has otherwise been well. Today, the child´s fever finally gets better and he develops maculopapular rash on extremities and face. Physical examination reveals a mildly uncomfortable child with temperature of 39°C (100.4°F). There is diffuse maculopapular rash on the trunk, as well as on the face and limbs. There are no pustules or excoriations. Which of the following pathogens is the most likely cause of this child´s symptoms?
Correct
Incorrect
Explanation:
A maculopapular rash that spreads from the trunk to the extremities and is particularly associated with high fevers should prompt one to think of roseola infantum, a common childhood disease. The causative agent of this condition, which is also known as exanthem subitum, is usually human herpesvirus 6 (HHV6), although HHV7 has been implicated.
Measles is rare because of near-universal vaccination and usually begins s a rash that starts at the heed and spreads toward the trunk, not vice versa. A description of Koplik spots, which are whitish elevations on an erythematous background, will usually be provided because the lesions are pathognomonic of measles.
Parvovirus B19 causes erythema infectiosum, which is characterized by a malar, erythematous rash often associated with high fevers. It starts on the face and often presents as a “slapped-cheeks´ rash. It does not start on the trunk and spread outward.
Rubella is another pediatric exanthem that classically begins on the face and spreads down the body. There is often a prodrome of lymphadenopathy and a low-grade fever, but not high, spiking fevers that can last for 2 weeks.
Varicella causes chickenpox. Vesicular lesions on an erythematous base, described s dew drops on a rose petal, and multiple excoriations are common findings in varicella infection. -
Question 60 of 100
60. Question
1 pointsAn 11-year-old girl has a sore throat since 3 days. There is redness and edema of the pharynx and enlarged red tonsils with spots of white exudate. She also has tender, enlarged anterior cervical lymph nodes. Which of the following is the best next in management?
Correct
Incorrect
Explanation:
This patient has presented with pharyngitis. Although the majority of pharyngeal infections are from viruses, the most important cause in this group is group A streptococci (Streptococcus pyogenes). This is important because if undiagnosed or untreated, it can progress to rheumatic fever or glomerulonephritis.
Rapid strep test is the best initial test because it is between 60 and 100% sensitive and is 95% specific. This means that a positive rapid strep test can be considered equivalent to positive culture and thus the patient can be treated immediately with penicillin. A negative rapid strep test, however, should be confirmed with a culture to bet certain that the patient does not have S. pyogenes Erythromycin is given to those patients who have a positive rapid strep test or a positive culture and who are allergic to penicillin. It is not the best next step in this case because a diagnosis of streptococcal pharyngitis has yet to be made.
Penicillin is used to treat patients with pharyngitis caused by S. pyogenes that has been confirmed by either a positive rapid strep test or a positive culture.
Pharyngeal culture is not the best next step in managing this case. Pharyngeal culture is performed after a negative rapid strep test to confirm that the patient does not have S. pyogenes; Urinalysis is not the best next step in management. Patients with streptococcal pharyngitis are at risk of developing glomerulonephritis, but this diagnosis has yet to be confirmed and nothing is mentioned in the question stem that would warrant urinalysis over a rapid strep test.
Blood culture is in no means equal to rapid strept test which allows quick evaluation. -
Question 61 of 100
61. Question
1 pointsA 6 month old white has “blisters” in his diaper area. You find large bullae filled with cloudy, yellow fluid. Some of them have ruptured and the bases are covered with a thin crust. Which one of the following is most appropriate in the management of this condition?
Correct
Incorrect
Explanation:
Bullous impetigo is a localized skin infection characterized by large bullae; it is caused by a group 2 phage type of staphyococcus aureus. Cultures of fluid from an intact blister will reveal the causative agent. Ordinary penicillin is usually effective in streptococcal impetigo but early treatment does not appear to lessen the occurrence of acute glomerulonephritis.
-
Question 62 of 100
62. Question
1 pointsA 10 year old complains of “head congestion” along moderate malaise and a low-grade fever for 7 days. He has had a thick discolored nasal discharge for the last 2 days. Which one of the following is correct?
Correct
Incorrect
Explanation:
Clinical diagnosis of bacterial sinusitis requires the following: Prolonged nonspecific upper respiratory signs and symptoms (i.e. severe upper respiratory tract signs and symptoms (i.e., fever 30˚C, facial swelling, and facial pain). This individual does not meet these criteria, so antibiotics should not be used at this time. Although some believe that mucopurulent rhinitis (thick, opaque, or discolored nasal discharge) indicates the presence of bacterial sinusitis, this sign should be recognized as part of the natural course of a nonspecific, uncomplicated viral upper respiratory infection (URI). Sinus radiographs can demonstrate thickened mucosa, infundibular occlusion, and occasional air-fluid levels in uncomplicated viral URI.
-
Question 63 of 100
63. Question
1 pointsWhich of the following causes Erythema infectiosum (fifth disease)?
Correct
Incorrect
Explanation:
Erythema infectiosum is characterized by a prodomal illness usually consisting of malaise, pharyngitis, and low-grade fever, followed by the appearance of a “slapped-cheek” rash. The cause of erythema infectiosum has been identified as parvovirus B19.
-
Question 64 of 100
64. Question
1 pointsIn a day-care center, two children develop systemic Haemophilus influenzae type b infections within the same month. You recommended prophylaxis with which one of the following for all children and staff in the classroom?
Correct
Incorrect
Explanation:
Whereas many antibiotics temporarily suppress nasopharyngeal colonization by Haemophilus influenzae type b, only rifampin is effective in eradicating the organism. It should therefore be administered to all attendees and staff of a day-care facility in which two or more children have been diagnosed with disease by H. influenzae, regardless of previous, immunization status. The patients should also receive rifampin before returning to the center. Prophylaxis after a single case is controversial.
-
Question 65 of 100
65. Question
1 pointsA 12 year old child has had several small, circular lesions on the left side of his lower forehead and nose, extending to the tip of his nose for 3 days. On examination you note grouped vesicles, several of which are scabbed. Burning pain in the area of the lesions is also present. He had a sore throat with a fever 5 days ago, but is now improved. His stepfather reports he is up-to-date on immunizations, but a specific immunization record is not available. His history and examination are otherwise unremarkable. The patient is most likely suffering from which disease?
Correct
Incorrect
Explanation:
Clustered circular lesions with accompanying dysesthesia in a dermatome are cahracteric of herpes zoster, which may occur after a stressful event or infection in both children and adults. For herpes zoster to occur there must be a previous primary varicella infection or immunization. Herpes zoster is less likely to be associated “with significant postherapeutic neuralgia in children than in adults.
-
Question 66 of 100
66. Question
1 pointsA 3 year old child presents with fever, prostration, and nuchal rigidity. He has received his primary immunizations but the pneumococcal conjugate vaccine has not been given. The cerebrospinal fluid (CSF) is cloudy, and microscopic examination shows innumerable neutrophils. The CSF protein is increased, and glucose is decreased. The most likely causative agent is which of the following?
Correct
Incorrect
Explanation:
Meningitis in newborns usually results from an infection of the bloodstream (sepsis). The infection is typically caused by bacteria acquired from the birth canal, most commonly group B streptococci, Escherichia coli, and Listeria monocytogenes. Older infants and children usually develop infection through contact with respiratory secretions from infected people. Bacteria that infect older infants and children include Streptococcus pneumoniae and Neisseria meningitidis. Haemophilus influenzae type b was the most common cause of meningitis, but widespread vaccination against that organism has now made it a rare cause.
Older children and adolescents with meningitis typically have a few days of increasing fever, headache, confusion, and a stiff neck. They may have an upper respiratory tract infection that is unrelated to the meningitis. Newborns and infants rarely develop a stiff neck and are unable to communicate specific discomfort. These younger children become fussy and irritable (particularly when they are held) and stop feeding, important signs that should alert parents to a possibly serious problem. Sometimes newborns and infants have fever, vomiting, or a skin rash One third have seizures. -
Question 67 of 100
67. Question
1 pointsA 13-year-old is brought to the physician with fever and sore throat. Her mother claims that she has been “lethargic” for the past week. Physical examination reveals lymphadenopathy and splenomegaly. The physician performs a monospot test, which reveals positive heterophil agglutination. A blood smear is taken and the predominant cell type seen is
Correct
Incorrect
Explanation:
The positive monospot test indicates atypical lymphocytes that this is infectious mononucleosis as a result of Epstein-Barr infection. Blood smears of EBV mononucleosis reveal a number of atypical lymphocytes.
-
Question 68 of 100
68. Question
1 pointsAll of the following perinatal infections can cause mental retardation EXCEPT
Correct
Incorrect
Explanation:
Perinatal hepatitis B is not associated with mental retardation. Rubella, cytomegalovirus, Toxoplasma gondii, and Herpes simplex are all part of the TORCH syndrome of perinatal infections, all of which include mental retardation as a major manifestation.
-
Question 69 of 100
69. Question
1 pointsA 4-year-old boy presents two days after stepping on a nail that pierced his sneaker. The puncture wound is erythematous and is draining a foul-smelling discharge. Which organism is responsible?
Correct
Incorrect
Explanation:
Pseudomonas infection is characterized by a foul-smelling discharge. This bacterium has been readily cultured from sneakers, and wounds that occur through sneakers should be considered to be infected by pseudomonas until proven otherwise.
-
Question 70 of 100
70. Question
1 pointsWhich of the following is NOT a common cause of neonatal meningitis?
Correct
Incorrect
Explanation:
Neisseria meningitidis is not a common cause of neonatal meningitis, while all of the others are.
-
Question 71 of 100
71. Question
1 pointsA 12 year old febrile boy presents with severe right lower quadrant pain that is interpreted as acute appendicitis. Joint pain is also present. During laparotomy the appendix is normal but the mesenteric lymph nodes are markedly enlarged and contain focal areas of microabscess formation on cut section. What is the likely way by which this pathogen was acquired?
Correct
Incorrect
Explanation:
Yersinia enterocolitica is the pathogen producing this clinical syndrome. Yersinia is transmitted to patients via the oral route most commonly from contaminated pork or milk, and, on occasion, from blood transfusion. It is associated with cooler climates and is a cause of pseudoappendicitis in children. Arthropod vectors are not involved in transmission of Y. enterocolitica. Arthropods such as ticks transmit tularemia, Rocky Mountain spotted fever, and Lyme disease. Insects such as mosquitoes transmit viral encephalitides and malaria. Contaminated ground beef is involved in the transmission of enterohemorrhagic Escherichia coil (EHEC), but is not a common source of infection with Y. enterocolitica. Dog bite is not involved in the transmission of Y. enterocolitica. Dogs may transmit rabies Capnocytophaga or Pasteurella. Endogenous transmission is important in the formation of intestinal abscesses by anaerobic normal flora organisms, but is not believed to be involved with Y. enterocolitica transmission.
-
Question 72 of 100
72. Question
1 pointsA child is referred to a hematology clinic after several visits to the emergency department with severe pain in her fingers and toes, requiring treatment with morphine. The child has fever on examination and a bone marrow aspirate contains no erythroid precursor cells. Which virus infected this girl?
Correct
Incorrect
Explanation:
Parvoviruses are small single-stranded DNA viruses of which only serotype B19 is pathogenic for humans. This virus causes three distinct syndromes: a childhood febrile rash known as erythema infectiosum (Fifth disease); aplastic crisis in individuals with chronic hemolytic diseases (sickle-cell anemia thalassemia, etc.); and congenital infections can present as stillbirth hydrops fetalis (analogous to severe Rh incompatibility), or severe anemia. Coxsackie viruses usually cause cold like illness but can cause herpangina, myocarditis, and meningitis. Echoviruses can infect a variety of organ systems (gastrointestinal, CNS, eyes, heart, respiratory, skin), but are not a cause of aplastic crises. Hepadnavirus is the causative agent of hepatitis B. Herpes viruses cause a variety of acute to chronic infections including herpes simplex types I and II, chickenpox, chronic herpes zoster, CMV infection, and Epstein-Barr virus infections.
-
Question 73 of 100
73. Question
1 pointsA 6-year-old has a low-grade fever and a rash that has been present for the last 36 hours. Examination shows faint generalized scarlatiniform eruption. Conjunctivae and oral mucosa are spared. A furuncle on the child´s left gluteal region is present. The skin is diffusely tender to palpation and has a “cigarette paper-like” appearance. Gentle lateral traction on the skin causes sloughing. Which of the following has a pathogenesis most similar to the disease elaborated?
Correct
Incorrect
Explanation:
This patient is suffering from staphylococcal scalded-skin syndrome (SSSS). The classic clinical presentation of SSSS is a child with a mild fever and a diffuse, faintly erythematous eruption. The eruption typically progresses to bulla formation and sloughing within 1 to 2 days. The mucosal surfaces are classically spared due to the higher concentration of desmoglein-3 in these sites.
This distinction helps to differentiate SSSS from toxic epidermal necrolysis or Stevens-Johnson syndrome. Nikolsky sign, as described in the question stem, is positive when lateral traction on the skin with the examiners finger causes sloughing. An infectious nidus, from where S. aureus is able to multiply and produce toxin, can often be observed as described in the question stem. SSSS is a toxin-mediated disease caused by staphylococcal exfoliative toxin-B, a form that selectively cleaves desmoglein-1, which is an essential component of the desmosomes that hold keratinocytes together. This pathogenesis is very similar to that of Bullous impetigo, where the production of exfoliative toxin-A causes local bulla formation at the site of a superficial cutaneous infection. Exfoliative toxin-A also cleaves desmoglein-1. Acute generalized exanthematous pustulosis is a widespread pustular disease that occurs in response to an offending medication, typically antibiotics. Epidermal sloughing is not a common feature, but fever is typically observed.
Epidermolysis bullosa acquisita is an acquired form of epidermolysis bullosa caused by autoantibodies against type VII collagen. Patients develop tense, non flaccid, non inflammatory, bullae at sites of repeated trauma such as the palms and soles. Toxic epidermal necrolysis and Stevens-Johnson syndrome are variants of the epidermal necrolysis syndromes. In these diseases, mucosal surfaces are always affected, most commonly early in the disease. The pathophysiologic of these conditions involves a CD8+ cytotoxic T lymphocyte-mediated induction of epidermal cell death through a Fas/FasL-dependent mechanism. These diseases usually occur in response to medication, and are histologically defined by full- thickness death and detachment of the epidermis. SSSS causes acantholysis within the epidermis, not epidermal cell death. -
Question 74 of 100
74. Question
1 pointsA 4-year-old has weakness and malaise since 1 month. He feels stiff. He now refuses to eat and is not gaining weight. He also complains of feeling dizzy and belly pain. Immunizations are all up to date. He is afebrile and his blood pressure is 80/48 mm Hg. He is found to have orthostatic hypotension. A stimulation test demonstrates a suboptimal response to cosyntropin. An assay for very long-chain fatty acids demonstrates a high level in the child´s blood. Which of the following is characteristic of this diagnosis?
Correct
Incorrect
Explanation:
This patient has adrenoleukodystrophy, an X- linked disease that results from mutations that prevent transport of very long-chain fatty acids (VLCFA) into peroxisomes, thereby preventing beta-oxidation and breakdown of VLCFA. Accumulation of abnormal VLCFA in effected organs (neurons and the adrenal cortex) is thought to be the underlying cause of the disorder. Patients thus present with:
Neurologic deficits: begins with weakness and spasticity; proceeds to dementia, blindness, and quadriparesis.
Primary adrenal insufficiency: hyponatremia, postural hypotension, hyperkalemia, and lack of response to the cosyntropin stimulation test Glycerol trierucate has not been shown to slow the progression of disease once onset has been documented. Overall it is not effective, although it may delay the onset of symptoms. Possible new therapies include lovastatin and 4-butyric acid analogues, which induce a protein capable of partially taking over the role of the defective peroxisomal membrane protein. Magnetic resonance of the brain shows lesions throughout the white matter, so it is unusual to have normal findings on MRI. Neurologic Findings may precede the diagnosis of adrenal insufficiency; however, in the majority of cases, adrenal disease is detected before neurologic findings. The responsible gene is a peroxisomal membrane protein, not mitochondrial membrane protein. -
Question 75 of 100
75. Question
1 points
A 17-year-old has had a painless lesion (pictured above) for the past 10 days. He does not use protection with any of his sexual partners. He has been treated for gonorrhea and genital warts. Hepatitis B serology show antibodies to hepatitis B surface antigen and core antigen. There is no hepatitis B surface antigen present. Which of the following statements is most correct?Correct
Incorrect
Explanation:
This patient has a syphilitic chancre and should be treated with benzathine penicillin G, 2.4 million units intramuscularly in a single dose. Because sexual partners might be infected even if they are seronegative, presumptive treatment should be given to those who were exposed within 90 days.
Benzathine penicillin G, 7.2 million units total administered as three doses is incorrect because this is the appropriate therapy for late latent syphilis or latent syphilis of unknown duration. Latent syphilis is characterized by seroreactivity without evidence of disease. Late latent syphilis is typically defined as latent syphilis that has been present for more than 1 year.
The patient in this case has a chancre, which represents a primary infection and not a latent infection. Because of his history of sexually transmitted disease (STD) he should be encouraged to have an HIV test Because this patient´s serology shows that he has a distant, resolved hepatitis B infection (meaning that he no longer has the disease), it is not necessary for his current sexual partners to receive post exposure prophylaxis against hepatitis B. In warts, treatment may reduce the infectivity but does not typically eradicate the infection -
Question 76 of 100
76. Question
1 pointsA 2-year-old boy has two small, fluid-filled lesions on his arm. They broke open with a little pressure. In 3 days, he then developed a fever to 102.2 F (39.0C), irritability malaise, and tenderness of the skin. Scarlatiniform erythema is present especially around the boy eyes, mouth, and antecubital fossa. The physician also saw two bullae on the child´s right leg. When he pushes down on the skin and moves his finger, the physician sees that there is a separation of the epidermis. Which of the following is the MOST appropriate treatment?
Correct
Incorrect
Explanation:
The diagnosis is staphylococcal scalded skin syndrome (SSSS) and it occurs in children under 5 years of age. The beginning of the illness usually consists of constitutional symptoms and skin tenderness, followed by a diffuse scarlatiniform erythema. This is often brighter in the flexural areas and around orifices, especially the mouth. There may also be flaccid blisters and erosions. Under gentle tangential pressure on any area of the skin, there may be separation of the epidermis. This is known as Nikolsky sign and may be seen in toxic epidermal necrolysis (but only at sites of erythema). As large areas of epidermis denude, most skin is exposed, which may become secondarily infected and lead to significant fluid and electrolyte losses. This phase starts from 2 to 5 days after the initial erythema and the disease resolves within 2 weeks without scarring. There is no involvement of the intraoral mucosa; however, the lips may be involved and there may be bilateral purulent conjunctivitis. The disease is caused mostly by phage group 2 staphylococci with hematogenous spread. The bacteria produce epidermolytic or exfoliative toxins A or B. Intact bullae are sterile, which will differentiate from Bullous imetigo. Skin biopsy will show the subcorneal, granular split of the epidermis. With diffuse involvement, an intravenous penicillinase-resistant penicillin such as nafcillin or oxacillin should be administered along with fluids. The skin is also moistened and cleaned with isotonic saline or a Burrow solution. An emollient adds lubrication and aids in comfort. Topical antibiotics are not needed.
Oral amoxicillin would not be effective against staphylococci, and in this case an intravenous antibiotic would be warranted.
Intravenous ampicillin is not penicillinase resistant. Ceftriaxone is a third-generation cephalosporin and as such would not be expected to have much gram-positive coverage.
Ciprofloxacin is contraindicated in children due to potential of cartilage damage. Preparations may be used topically while the macrolides such as erythromycin may be effective against a staphylococcal infection, this case warrants intravenous antibiotics.
Erythema toxicum is a self-limited disease and does not really require treatment. -
Question 77 of 100
77. Question
1 pointsA 2-year-old girl lives with her grandmother as the mother is an addict and father is dead. She was below the 5th percentile for weight last year and she has not had any immunizations since then. She stands independently but cannot walk without assistance. Temperature=37.8 °C and pulse= 115/min. She has oral thrush and chronic otitis media along submandibular, cervical, axillary, and inguinal lymph nodes enlargement and firmness. Diagnose:
Correct
Incorrect
Explanation:
This patient presents with failure to thrive, developmental delay, fever, oral thrush, an ear infection, and generalized lymphadenopathy. She has a positive risk factor for human immunodeficiency virus (HIV) infection in that her mother has a substance abuse problem. Together, these findings most likely indicate this patient has acquired immune deficiency syndrome (AIDS). AIDS may be caused by the HIV type 1 or type 2. In children, infection with HIV type 2 is rarely found. Most HIV-infected children are born in developing countries and the majority of cases are acquired via vertical transmission from mother to child. Breastfeeding is also a significant route of transmission. Cesarean section combined with prenatal, intrapartum, and neonatal zidovudine therapy significantly reduces transmission of the virus. Sexual contact is a major route of transmission in adolescents. Three patterns of disease have been described in children:
1) HIV-infected newborns with rapid onset of symptoms and acquired immune deficiency syndrome in the first few months of life
2) Prenatally infected newborns with slower progression of disease
3) Long-term survivor with minimal or no progression of disease and normal CD4 counts.
Clinical status and degree of immunologic impairment are used to classify HIV in children. In the majority of infants, the physical examination at birth is normal. However, over a period of time the infant may develop failure to thrive. Problems such as lymphoid interstitial pneumonia, chronic otitis media, persistent diarrhea, hepatosplenomegaly, recurrent bacterial sepsis, and candidiasis may be seen. Older children have similar findings as adults. Detection of antibodies is performed by enzyme-linked immunosorbent assay (ELISA) and Western blot analysis. Passively acquired maternal antibodies last up to 15 months. All infants born to HIV-positive mothers have positive antibody tests at birth because of passive transfer of maternal antibodies. HIV DNA polymerase chain reaction is the preferred virologic assay for infants in developing countries. HIV culture is not recommended for infants less than 1 month of age because of potential for false positive results. HIV disease can be excluded in children over 18 months of age who have at least two HIV antibody tests that are negative, absence of hypogammaglobulinemia, and no clinical evidence of HIV disease. Treatment with antiretroviral therapy in children infected with HIV depends on the viral load, CD4 count and percentage, and clinical condition. Decisions regarding therapy should be made in consultation with an expert in pediatric HIV disease. Opportunistic infection should be treated as the child develops and nutritional support should be given.
Disseminated herpes simplex virus infection is most commonly seen in children between 6 months and 3 years of age. It starts with a severe herpetic gingivostomatitis followed by dissemination to the liver and other internal organs. Massive viremia may ensue with gastroenteritis, encephalitis, and hepatic and adrenal dysfunction.
Death frequently occurs during the phase of massive viremia.
Umbilicated vesicles that evolve into necrotic ulcers may be seen on the skin and mucosal surfaces. Disseminated herpes simplex virus infection may occur in premature newborns, malnourished infants, or patients treated with immunosuppressive drugs, as well as in Wiskott-Aldrich syndrome. Treatment is with intravenous acyclovir.
Infectious mononucleosis is characterized by fever, adenopathy, splenomegaly, lymphocytosis with atypical lymphocytes, and evidence of acute infection by Epstein-Barr virus. After an incubation period of 3 to 7 weeks, bilateral enlargement of the cervical lymph nodes and, at times, of the axillary and inguinal lymph nodes develops with accompanying fever as high as 40 °C (104 °F), malaise, and headache. Pharyngitis is the most frequent sign, with hyperplasia of the pharyngeal lymphoid tissue. Diagnosis is confirmed with the monospot test. Treatment is supportive with rest and avoidance of contact sports until the splenomegaly has resolved.
Streptococcal pharyngitis presents with sudden-onset fever and sore throat. Physical examination reveals very enlarged, erythematous, edematous tonsils covered with purulent exudate. The submandibular lymph nodes may be enlarged and tender. Diagnosis is confirmed with a pharyngeal bacterial culture. The treatment of choice is penicillin.
Systemic candidiasis is a severe, destructive, disseminated disease that occurs in the setting of compromised host defenses.
Hemorrhagic pustules, necrotic ulcers, and deep abscesses are seen on the skin and mucosal surfaces. Other clinical findings depend on organ involvement and may include gastrointestinal bleeding, fever of unknown origin, pulmonary infiltration, endocarditis, renal failure, meningitis, osteomyelitis, endophthalmitis, peritonitis, or a disseminated maculopapular exanthem. Diagnosis is made by demonstrating microorganisms or a positive culture from tissue or fluids that are normally sterile. Mortality is approximately 40%. The standard treatment is amphotericin B, although multiple other systemic antifungal agents are available on the market today. -
Question 78 of 100
78. Question
1 pointsA 4 months infant presents with failure to thrive, progressive muscular weakness, and poor head control. The mother says that she typically feeds the baby soy based formula sweetened with honey. This child´s symptoms are most likely caused by ingestion of
Correct
Incorrect
Explanation:
This baby has infant botulism (floppy baby syndrome), which is due to germination of Clostridium botulinum spores in the baby´s gastrointestinal tract. This disorder is most common in children under the age of 6 months. Older children and adults do not appear to be vulnerable to this form of botulism, but are susceptible to botulism caused by ingestion of preformed toxin. Floppy baby syndrome is acquired following the ingestion of spores of the organism. Dipicolinic acid and calcium dipicolinate are unique to bacterial spores. The former is thought to be responsible for the heat resistance of the spore, while the later helps to stabilize the DNA. A gram positive vegetative bacillus describes Clostridium botulinum, but it is not ingestion of the vegetative bacteria that causes this condition. A neurotoxin encoded on a nonconjugative plasmid describes tetanospasmin, which is produced by Clostridium tetani. This toxin causes a rigid paralysis due to the inhibition of glycine and GABA at inhibitory synapses.
A neurotoxin encoded in a phage genome would describe the botulinum toxin erythrogenic exotoxins of Streptococcus pyogenes and the diphtheria toxin. It is not ingestion of the preformed toxin that causes floppy baby syndrome; it is ingestion of spores. A 150,000 molecular weight progenitor neurotoxin is a description of both the botulinum toxin and the tetanospasmin, but it is not the toxin that is ingested to cause floppy baby syndrome, but the spore of the bacterium. -
Question 79 of 100
79. Question
1 pointsA 6 year old boy presents to a pediatrician because of a severe sore throat that is accompanied by a rash. Examination reveals pharyngeal injection, swollen, bright red tonsils with discrete white spots on the surface, and enlarged anterior cervical lymph nodes. Skin has a diffuse, erythematous, symmetrical rash that blanches on pressure, has slightly rough sandpaper like texture, and is most prominent on the neck, chest, and folds of the axilla, elbow, and groin. In which of the following way would the strain of the organism that produces the child´s disease most likely to differ from a less pathogenic strain of the same organism?
Correct
Incorrect
Explanation:
The rash described is that of scarlet fever, which can be a complication of streptococcal sore throat (as in this question) or streptococcal wound infection. In past centuries, scarlet fever was a major childhood disease that often caused death, but it has become much less common (possibly, as a result of early antibiotic therapy) and is now only a rare cause of death. Strains of streptococci that cause scarlet fever are beta hemolytic group A streptococci that have been infected by a phage. The precise role of the phage is still unclear but may involve up regulation of toxin producing genes because some of the streptococcal strains that do not produce scarlet fever still carry genes for toxin production. Lipopolysaccharide is found in gram-negative organisms. Spore formation is a feature of Bacillus and Clostridium. Mycoplasma are organisms that lack cell walls. Lecithinase is an enzyme produced by Clostridium.
-
Question 80 of 100
80. Question
1 pointsA 3 year old otherwise healthy child presents with a rash. According to mother the rash appeared recently, but the boy has been eating well and has been afebrile. Examination reveals multiple umbilicated, flesh colored papules on the patient´s trunk. A characteristic of the most likely infectious agent is which one of the following?
Correct
Incorrect
Explanation:
The lesions are characteristic of molluscum contagiosum, which is a typically benign and self limited condition caused by a poxvirus. The disease can be transmitted either sexually or through direct nonsexual contact. Patients with advanced HIV infection may develop a severe, generalized, and persistent eruption often involving the lace and upper body. Molluscum contagiosum virus is a poxvirus that reproduces in the cytoplasm and produces intracytoplasmic inclusion bodies. Capturing a nuclear membrane envelope is characteristic of Herpesviridae. This family includes herpes simplex 1 and 2, cytomegalovirus, Epstein Barr virus varicella zoster, herpes 6, and herpes 8. None of these agents cause umbilicated warts, although rashes may occur. Herpes simplex causes vesicular lesions on the mucosae, and varicella causes an asynchronous pruritic rash (chickenpox) or a unilateral vesicular rash that follows a dermatome (shingles). Production of synchronous rash describes smallpox. This virus has been extinct since 1977 and produces hemorrhage into the dermis not wart formation. Owl´s-eye inclusion bodies are characteristic of cytomegalovirus but not molluscum contagiosum. Molluscum contagiosum produces intracytoplasmic inclusion bodies (Guarnieri bodies) that function as factories of DNA synthesis in the cytoplasm of infected cells. Some serotypes of human papilloma virus are pre-neoplastic. The warts of human papillomavirus have a dense fibrous core and are not umbilicated, as described in this question stem. The pre-neoplastic serotypes are 16, 18, and above. -
Question 81 of 100
81. Question
1 pointsThe mother of a 3-year-old child who is infected with HIV would like to enroll her child in a local child care center. The child is able to speak in sentences of 3 to 4 words, to follow commands and feed himself, to draw a circle, and is currently practicing toilet-training. Of the following, which one is sufficient to exclude the child from day care?
Correct
Incorrect
Explanation:
Other than through blood exposure, HIV has not been transmitted through other types of daily contact that occur in daycare centers, including saliva and tears. Children who have HIV should therefore not be excluded from child care or school settings.
Even children who have aggressive behavior or a history of nosebleeds should not be excluded based on those reasons.
Urine and stool do not provide a route of transmission for HIV, and therefore not being toilet-trained also does not provide a reason for exclusion. Disclosure of HIV status to the child care center or school is not required by law. -
Question 82 of 100
82. Question
1 pointsA previously healthy 3-week-old infant is irritable and restless for the past 3 hours along incessant crying during this time, as well poor feeding. The baby also vomited three times and passed loose stool twice. She is currently being bottle-fed and has been developing well. On examination, the patient appears unwell, is crying, and has decreased muscle tone. Temperature is 38 C (101°F). Examination further reveals respiratory distress with grunting, retraction, and bradycardia. There is no evidence of a rash. Head and neck examination reveal full fontanelle and normal neck flexion. What is the most likely cause of this patient´s condition?
Correct
Incorrect
Explanation:
This neonate is most likely suffering from sepsis with or without meningitis, as indicated by respiratory distress (grunting, retractions), bradycardia, temperature instability, poor feeding, lethargy/irritability, vomiting, and diarrhea. Note that the findings of a full (normal), rather than a bulging, fontanelle and of normal neck flexion do not rule out meningitis as the diagnosis because at the time of initial presentation of neonatal meningitis, only 25% of patients present with a bulging fontanelle and only 15% with nuchal rigidity. The clinical presentation of neonatal meningitis is often indistinguishable from that of neonatal sepsis without meningitis, so the diagnosis of sepsis with or without meningitis is confirmed by blood and CSF findings. One of the most common causes of meningitis among neonates and infants up to the age of 3 months old is Listeria monocytogenes. Other agents responsible for meningitis in this age group include Escherichia coli and Group B Streptococcus. Neonates and the elderly have decreased T-cell immune function; therefore, Listeria is more common in the very young and the very old.
Borrelia burgdorferi is not as common as Listeria among neonates and infants. B. burgdorferi is the etiologic agent behind Lyme disease, a zoonotic disease transmitted by ticks. The question will usually give a clue, such as a tick-bite or an annular red rash (the erythema migrans rash that occurs at the site of the tick bite), or a patient from Connecticut, Massachusetts, or New York.
Cryptococcus neoformans is a fungus that occurs primarily in patients who are immunocompromised, such as those who are HIV-positive. . Cryptococcus is usually associated with those HIV-positive patients who have profound decreases in their T-cell counts to levels less than 100 cells.
Mycobacterium tuberculosis is incorrect; there is nothing in the question stem that points toward this etiologic agent (such as a history of TB, cough, weight loss, or night sweats). It is not as common as Listeria among neonates and infants.
Neisseria meningitidis is the most common cause of meningitis among adolescents and young adults. It is spread by respiratory droplets, and results in the characteristic petechial rash. It is not the most common cause of meningitis among neonates and infants.
Streptococcus pneumoniae is the most common cause of meningitis overall; however, it is not the most common cause of meningitis among neonates and infants. Also note that the respiratory distress in the patient is due to neonatal sepsis, and not necessarily to pneumonia.
Treponema pallidum is the causative agent behind syphilis. It is not the most common cause of meningitis among neonates or infants. Congenital syphilis is divided into early and late stages.
Early-stage manifestations appear before 2 years of age and include fever, anemia, failure to thrive, a maculopapular rash, snuffles, hepatosplenomegaly, periostitis of the long bones, and thrombocytopenia. Late-stage manifestations are skeletal and include saber shin, Hutchinson teeth, saddle nose, and knee synovitis.
Rotavirus is the most common cause of gastroenteritis in children. It presents with diarrhea, vomiting, and fever. It rarely affects infants less than 6 months of age with severe symptoms. It can never cause sepsis. The present patient is clearly septic. -
Question 83 of 100
83. Question
1 pointsA 5-year-old boy has a 3-week history of diarrhea, abdominal pain, and tenesmus. There is blood in the toilet after he goes to the bathroom. Temperature is 38.9°C (102.0°F), blood pressure is 90/60 mmHg, pulse is 130/min, and respiration is 20/min. He has diffuse abdominal pain. During the examination, he passes a very foul-smelling stool that is sent for analysis. What would be the next best step in management?
Correct
Incorrect
Explanation:
The patient described in the question is infected with Entamoeba histolytic the symptoms of which can range from completely asymptomatic (cyst shedding) to intestinal amebiasis to extraintestinal disease. Typically a 2-week incubation period is present after which diarrhea can present, sometimes progressing to colitis in children younger than age 5 years. A chronic form of amebic colitis can develop, mimicking inflammatory bowel disease. Disseminated amebiasis can occur and is a very serious complication of the infection, resulting in liver abscess formation. That is the reason an abdominal ultrasound must be done to evaluate for a hepatic abscess.
Treatment is metronidazole, followed by paromomycin, or iodoquinol.
A barium enema is contraindicated in a patient who has colitis because of the increased risk for developing toxic megacolon or perforation.
Colonic biopsy may show the amebic organisms, but it is not the next most appropriate step in management.
Gallium scan may help in finding the liver abscess, but it is not more cost-effective than an ultrasound.
Liver function tests would be normal, because the abscess formation does not destroy the actual liver cells. -
Question 84 of 100
84. Question
1 pointsAn 8 year old boy is taken to a pediatrician because of behavioral changes, mild intellectual deterioration, and “laziness.” He develops increasing clumsiness and periodic, involuntary, jerky movements every 3 to 6 seconds over the next several months. Optic atrophy is evident on funduscopic examination. Oligoclonal bands of IgG are present on CSF electrophoresis. EEG shows periodic discharges that are synchronous with the periods of myoclonus. Head CT scan shows low density white matter lesions and cerebral atrophy. The boy dies at the age of 10. The child´s condition was related to past infection with which one of the following?
Correct
Incorrect
Explanation:
The rare disease illustrated is subacute sclerosing panencephalitis(SSPE),which typically presents as in the question stem. SSPE appears to be due to a combination of persistent, possibly abnormal measles virus and to autoimmune damage caused by antibodies directed against the virus. Many patients developing SSPE have had measles at 2 years of age or younger; there is typically a six-year interval between measles infection and symptom development. A small proportion of cases have followed vaccination with live measles virus. Unfortunately, no effective therapy has been developed, although some experimental work with drugs such as isoprinosine shows some promise. Post infectious encephalomyelitis, rather than SSPE, can follow mumps or varicella. Latent infection with the varicella virus causes shingles (herpes zoster). Papilloma viruses are associated with warts. They do not usually infect the brain. Poliovirus causes gastrointestinal disturbances viremia, and paralysis.
-
Question 85 of 100
85. Question
1 pointsA 2 year old boy has surgery to correct a urinary tract obstruction. He develops a urinary tract infection postoperatively with an indwelling catheter in place. Urine culture grows out a non-lactose fermenting, oxidase-positive, gram-negative rod. What condition is also caused by the likely organism in this case?
Correct
Incorrect
Explanation:
Pseudomonas aeruginosa is a gram-negative rod. It can easily be distinguished from the family Enterobacteriaceae because Pseudomonas is oxidase-positive. It is an opportunistic pathogen that has an increased chance of causing urinary tract infections in patients who have indwelling catheters or who are on antibiotics. Pseudomonas is an agent of hot-tub folliculitis in normal patients using contaminated spas, and causes infections in burn patients and pneumonia in patients with cystic fibrosis. Esophagitis in AIDS patients can be caused by Candida albicans. This is a yeast that can cause urinary tract infections in poorly controlled diabetics because glucose in the urine enhances its growth. Neonatal septicemia is most frequently caused by Streptococcus agalactiae, Escherichia coli, or Listeria monocytogenes. Of these S. agalactiae is a gram-positive coccus and L. monocytogenes is a gram-positive rod, and although E. coil is a gram-negative rod it is oxidase-negative and lactose fermenting. E. coli is the most common cause of urinary tract infection overall. Pneumonia in AIDS patients is caused by a variety of organisms, including Histoplasma capsulatum, Coccidioides immitis, Blastomyces dermatitidis, and pneumocystis jiroveci. None of these is a gram negative bacillus. Staghorn calculi in the urinary tract are formed during infections with urease-positive organisms such as Proteus and Ureaplasma. Proteus is a gram-negative rod but is oxidase-negative, and Ureaplasma is a cell-wall less prokaryote related to Mycoplasma that would not stain at all with Gram stain. -
Question 86 of 100
86. Question
1 pointsA 14 year old girl living in a rural community has swollen, painful lymph nodes in her right axilla. Examination reveals multiple abrasions on her right arm with a papule associated with one of the abrasions. Which of the following is caused by the same infectious agent as the one causing this patient´s condition?
Correct
Incorrect
Explanation:
This patient has acquired cat scratch disease, which is caused by the gram negative rod, Bartonella henselae. Bartonella henselae and B. quintana are also responsible for the production of bacillary angiomatosis, which is a vascular proliferative disease involving skin, bone, and subcutaneous tissues in immunocompromised patients. Oroya fever is an acute febrile illness with severe anemia caused by Bartonella bacilliformis. Rat bite fever is caused by Streptobacillus moniliformis, which is a gram-negative bacillus unrelated to the genus bartonella. Trench fever is a febrile illness caused by Bartonella quintana which is transmitted by the human body louse. Verruga is the chronic cutaneous form of Oroya fever, which is caused by Bartonella bacilliformis.
-
Question 87 of 100
87. Question
1 pointsA 12-year-old boy has streptococcal pharyngitis. He is leaving for a summer camp in 2 days. In the past, he has had problem finishing the whole course of antibiotic treatment. He has eczema and asthma but is not all allergic to any medications. Which of the following is the best treatment for his streptococcal pharyngitis?
Correct
Incorrect
Explanation:
The treatment of choice for streptococcal pharyngitis is oral penicillin V for 10 days. However, the child in this clinical vignette is leaving town for a summer camp in 2 days. There is no way to monitor his compliance to the regimen if he is not with the family. Also, he has had a problem finishing his medicine in the past. A better regimen for him this time might be a single dose or an appropriate antibiotic agent among the above three choices, only benzathine penicillin G is the appropriate treatment for streptococcal pharyngitis. Benzathine penicillin G can be given as a single dose intramuscularly. It is a long-acting antibiotic and can complete the treatment. Intramuscular ceftriaxone does not effectively eradicate streptococcal infection in the throat Intramuscular procaine penicillin is a short-acting penicillin. One dose at does not eradicate streptococcal pharyngitis.
Oral treatment for streptococcal pharyngitis requires a total regimen of 10 days. Both erythromycin and penicillin V are appropriate drugs. But again, a single intramuscular antibiotic is a better treatment for this patient. -
Question 88 of 100
88. Question
1 pointsA 7-year-old has bumps and a rash on her body. The bumps began to 3 weeks ago and thay developed a surrounding rash and began to increase in number in a linear fashion. Neck, face, and trunk are affected by pink, glossy papules with umbilicated centers shown as follows. At some places, a surrounding zone of dermatitis around the papules can be seen and some papules do appear to be arranged in a linear fashion. Skin biopsy with virology reveals the presence of a poxvirus. What is the culprit?
Correct
Incorrect
Explanation:
This patient is suffering from a classic case of molluscum contagiosum (MC). MC is caused by the molluscum p. contagiosum virus, which is a member of the Poxviridae family of viruses; Poxviridae are large, boxcar-shaped, enveloped dsDNA viruses.
MC is a very common viral infection, with up to one-quarter of the population showing serologic evidence of past infection. This condition can affect children or adults and can be contracted following casual or sexual contact. The overwhelming majority of cases in children are not due to sexual abuse. The typical clinical presentation is with red-to- pink, glossy papules with indented or umbilicated centers. The eruption is often slightly itchy, and scratching causes spread of the infection through autoinoculation (Koebnerization), resulting in linear aggregations of papules. A mild eczematous eruption known as molluscum dermatitis can frequently form surrounding the papules.
The cowpox virus is the cause of milker´s nodules. It is also known as the vaccinia virus and was used as the first form of vaccination against smallpox. Historically, the fingers, hands, and forearms of people who handled infected cow udders developed solitary or multiple large vesicular lesions.
The Orf virus is a poxvirus that causes a disease known as Orf. Orf is a zoonosis contracted after contact with infected animals, typically livestock. The associated lesions typically are solitary or few and tend to affect the fingers.
The varicella zoster virus which causes chickenpox and shingles is a herpesvirus (HHV3). The eruption in chickenpox manifests as recurrent crops of lesions that resemble “dewdrops on a rose petal. The variola Vera virus which causes smallpox is a member of the Poxviridae family of viruses. Smallpox is a disease that is considered eradicated worldwide but is still of significance due to its potential for use in biologic warfare. The eruption is characterized by a single, diffuse crop of macules evolving to vesicles and ultimately pustules. -
Question 89 of 100
89. Question
1 pointsA 5-day-old has poor feeding and is irritable, crying all the time and jittery with irregular breathing over the last few hours. Hands and feet have turned blue. During delivery, the labor had been prolonged and the membranes had ruptured 12 hours before delivery. On physical examination, the infant has a rectal temperature of 35.6 o C (96.1 oF), pulse is 95/min, and respirations are 30/min. He is irritable and grunting during the exam. The anterior fontanelle is bulging and there is peripheral cyanosis. Which of the following is the most likely diagnosis?
Correct
Incorrect
Explanation:
This child has developed neonatal sepsis. Sepsis is a systemic response to infection. In newborns it can be classified as early-onset, occurring in the first week of life; and late-onset occurring between age 8 and 28 days. Risk factors for neonatal sepsis include maternal infection during pregnancy (urinary tract infection, chorioamnionitis), prematurity, and prolonged rupture of membranes. Common organisms causing, sepsis in the newborn are group B streptococci, Escherichia coli, and Listeria. Viral causes can be herpes simplex virus and enteroviruses. Signs and symptoms of neonatal sepsis are nonspecific, such as grunting, tachypnea, cyanosis, poor feeding, irritability, apnea, bradycardia, jitters, tremors, and seizures. Newborns do not always develop fever, and hypothermia may be a presenting sign. A bulging fontanelle may be palpated on physical exam, but there is rarely nuchal rigidity. Laboratory studies should include a complete blood count lumbar-puncture, and blood and urine cultures, as well as cultures of any visible lesions or drainage. A chest radiograph should be done to exclude pneumonia. Treatment consists of antibiotics that should be started empirically in expectation of culture and sensitivity studies.
Congenital rubella occurs in 80% of babies if the mother is infected during the first trimester. The most common manifestation is intrauterine growth retardation, followed by congenital cataracts, mental retardation, heart defects (patent ductus arteriosus, pulmonary stenosis), hepatosplenomegaly, deafness, and blueberry-muffin” lesions (foci of extramedullary hematopoiesis). Diagnosis is made by culture or IgM titers. Immunization of the mother prevents disease.
Congenital syphilis appears before 2 years of age and includes fever, anemia, failure to thrive, maculopapular, rashes snuffles, hepatosplenomegaly, periostitis of the long bones, and thrombocytopenia. It is a consequence of transmission from the mother within the first 4 months of pregnancy.
Congenital toxoplasmosis is less; common but more serious if the maternal infection happened in the first trimester of pregnancy and more common but less severe if the maternal infection occurs in the third trimester. It manifests with intracranial calcifications, intrauterine growth retardation, microcephaly, seizures, blindness, and hepatosplenomegaly. Diagnosis is made from cultures and determination of IgM titers. Treatment for the pregnant woman in the first trimester is spiramycin, and pyrimethamine and sulfonamide thereafter. The newborn is treated with pyrimethamine, sulfonamide, and leucovorin. If the neonate has chorioretinitis, steroids are administered with the goal of preventing blindness. Infection with toxoplasmosis is acquired through eating undercooked or raw infected meat or from handling infected cat feces. Neonatal herpes simplex virus infection is usually acquired during passage through in infected birth canal. Primary disease in the mother has a much higher transmission rate to the neonate than recurrent disease.
Delivery by cesarean section should be performed if there are active genital lesions in the mother. Infection may follow three general patterns: local (skin, eye, and mouth), appearing at 5 to 14 days; disseminated (pneumonia, shock, hepatitis), at 5 to 7 days; and central nervous system (lethargy, seizures), at 3 to 4 weeks.
Mortality is high with disseminated disease. Diagnosis can be made by clinical pattern, culture, polymerase chain reaction of the cerebrospinal Fluid, and demonstration of antibodies. Treatment of choice is acyclovir. -
Question 90 of 100
90. Question
1 pointsA 6-month-old infant has decreased activity and poor feeding over the past week. He has been constipated for the past 2 days. He has a very weak cry, poor muscle tone, and absent deep tendon reflexes. The rest of the examination is unremarkable. Which of the following is the most likely diagnosis?
Correct
Incorrect
Explanation:
Infant botulism results from the production of toxin after colonization of the gastrointestinal tract by Clostridium botulinum in young children aged 1-9 months. The most common source of the organism is the soil or, less frequently, honey. Nearly all cases are due to types A or B. The incubation period is usually between 18 and 36 hours. Short incubation periods are associated with more severe disease. The disease spectrum varies considerably but the most commonly recognized form is the “floppy baby syndrome: Initial symptoms are lethargy, diminished suck, constipation, weakness, feeble cry, and diminished spontaneous activity with loss of head control.
These symptoms are followed by extensive flaccid paralysis. The case fatality rate is only 1 %. The bulbar musculature is usually affected first. In older children, it results in diplopia, dysarthria, and dysphagia. Involvement of the cholinergic autonomic nervous system may result in decreased salivation, with dry mouth and sore throat, ileus, or urinary retention. Neurologic evaluation often shows bilateral paresis of the 6th cranial nerves, ptosis, dilated pupils with sluggish reaction, decreased gag reflex, or medial rectus paresis. These symptoms are followed by descending involvement of motor neurons to peripheral muscles, including the muscles of respiration. Patients are usually febrile with clear mentation. The most common cause of death is respiratory failure. The spectrum of disease is quite variable; some patients have mild disease, whereas others have severe paralysis requiring mechanical ventilation. Respiratory failure is the major risk, and patients must be monitored carefully with liberal use of ventilatory support. Toxins can be removed from the gastrointestinal tract with gastric lavage, cathartics, and enemas early in the course of disease. The trivalent antitoxin or type-specific antitoxin for types A, B, and E is usually given only to adults. Infants with botulism should not receive either antibiotic directed against C.
Botulinum or antitoxin, because most do extremely well with supportive care alone and it has been suggested that antibiotics may cause toxin release. Honey has been implicated as a vehicle for spores and should not be fed to infants younger than 1 year.
Congenital hypothyroidism, or cretinism, manifests as impaired development of the skeletal system and CNS. It is associated with severe mental retardation, short stature, coarse facial features, a protruding tongue, and umbilical hernia.
Guillain-Barre syndrome presents with weakness that develops symmetrically over several days. The weakness typically occurs first in the legs and ascends with time to involve the muscles of the trunk, intercostals, upper extremity, and neck. Muscles innervated by cranial nerves are also involved. Respiratory paralysis can lead to death within hours to days.
Myasthenia gravis is a disorder of the neuromuscular junction resulting in a pure motor syndrome characterized by weakness and fatigue, particularly of the extraocular, pharyngeal, facial, cervical, proximal limb, and respiratory musculature. Fifteen percent of infants born to myasthenic mothers have neonatal myasthenia gravis because of the transplacental passage of acetylcholine receptor antibodies. The condition completely resolves in weeks to months.
Vaccine-associated poliomyelitis is exceedingly rare; only eight or nine cases are reported yearly. Most symptomatic cases have nonspecific manifestations of infection illness is biphasic, and paralysis occurs in the second phase. Paralytic disease occurs with rapid onset, involving the cranial nerves, arms, and legs.
-
Question 91 of 100
91. Question
1 pointsAn infant presents to you 7 days after birth with bilateral purulent conjunctive discharge, erythema and swelling at the medial aspect of the eyes. Out of the following, which is the most likely diagnosis?
Correct
Incorrect
Explanation:
Neonatal conjunctivitis is purulent ocular drainage due to a chemical irritant or a pathogenic organism. Topical prevention is routine. Chlamydial ophthalmia usually occurs 5 to 14 days after birth. It may range from mild conjunctivitis with minimal mucopurulent discharge to severe eyelid edema with copious drainage and pseudomembrane formation. Follicles are not present in the conjunctiva, as they are in older children and adults.
In chlamydial ophthalmia, systemic-therapy is the treatment of choice ´because at least 1/2 of affected neonates also have nasopharyngeal infection and some develop chlamydial pneumonia. Erythromycin 12.5 mg/kg po q 6 h for 2 weeks is recommended. Efficacy of this therapy is only 80%, so a 2nd treatment course may be needed. -
Question 92 of 100
92. Question
1 pointsA 13 year old boy middle-school wrestler comes to you complaining of a recurrent painful rash on his arm. Several dry vesicles are present. Which is the most likely diagnosis?
Correct
Incorrect
Explanation:
The most common infection transmitted person to-person in wrestlers is herpes gladiatorum caused by the herpes simplex virus. Molluscum contagiosum causes keratinized plugs. Human papillomavirus causes warts. Tinea corporis is ringworm, which is manifested by round tm oval raised areas with central clearing. Mat burn is an abrasion.
-
Question 93 of 100
93. Question
1 pointsChildren under 1 year of age should not be given honey because of possible contamination with
Correct
Incorrect
Explanation:
The most common cause of infant botulism is ingestion of Clostridium botulium spores in honey.
-
Question 94 of 100
94. Question
1 pointsA 5-year-old has sudden development of fever, headache, muscle pain, and malaise. He also has a large swelling on the right side of his jaw lines and neck. Temperature = 39.3°C (102.8°F) and pulse = 110/min and oral mucosa shows erythema and edema surrounding the right Stensen canal along a tender, firm, non-mobile mass at right mandibular angle. Which complication is most likely?
Correct
Incorrect
Explanation:
This boy has developed mumps. The most common complication in prepubertal children is meningoencephalomyelitis due to spread of infection in the CNS. Mumps is a viral infection with a paramyxovirus that causes painful enlargement of the salivary glands, predominantly the parotid glands. Transmission of the mumps virus occurs via airborne droplets, direct contact and fomites contaminated with saliva. It is more common in the winter and spring. Outbreaks are related to lack of immunization. The treatment of mumps is supportive. This is a viral infection and not related to streptococcal meningitis.
Dacryoadenitis is a rare complication of mumps in which the lacrimal gland and duct are inflamed as a consequence of the viral infection.
Infertility is not a common complication of mumps, even with bilateral orchitis. Mumps oophoritis may develop in postpubertal females but it does not affect fertility.
Orchitis is an uncommon complication in prepubertal children, even though it is the most common complication in adult males. It may be unilateral or bilateral and is treated with support and bed rest. In 30% of cases it may be bilateral. Even in those cases, infertility is rare. -
Question 95 of 100
95. Question
1 pointsA previously healthy 2-month-old boy develops irritability and restlessness for the past 6 hours with incessant crying and poor feeding. He has decreased muscle tone. The fontanelle is bulging. CSF shows decreased glucose and increased proteins and Neutrophilia. What is the most likely cause of meningitis here?
Correct
Incorrect
Explanation:
One of the most common causes of meningitis among neonates and infants up to the age of 3 months is Group B Streptococcus (GBS), otherwise known as Streptococcus agalactiae. Other agents responsible for meningitis in this age group include Escherichia coli and Listeria monocytogenes. CSF findings indicate bacterial pattern.
Borrelia burgdorferi is not as common as GBS among neonates and infants. S. burgdorferi is the etiologic agent behind Lyme disease, a zoonotic disease transmitted by ticks. The question will usually give a clue, such as a tick bite or an annular red rash (the erythema migrans rash that occurs at the site of the tick bite).
Cryptococcus neoformans is a fungus that occurs primarily in patients who are immunocompromised, such as those who are HIV-positive. Although it is common among this patient population, it is not the most common agent to cause meningitis among this population. Cryptococcus is usually associated with those HIV-positive patients who have profound decreases in their T-cell counts to levels less than 100 cells.
Neisseria meningitidis is the most common cause of, meningitis among adolescents and young adults. It is spread by respiratory droplets and results in the characteristic petechial rash. It is less common among neonates and infants before 3 months of age.
Streptococcus pneumoniae is the most common cause of meningitis overall, but it is not the most common cause of meningitis among neonates and infants.
Cytomegalovirus causes meningitis in immunocompromised. -
Question 96 of 100
96. Question
1 pointsAn 11-day-old infant has 1 day old fever and poor feeding. He was full-term to a GBS-negative mother. Temperature = 39.2 °C (100.8°F). CSF shows increased protein and decreased glucose with Leukocytosis (Neutrophils=68%, Lymphocytes=30%, Bands=2%). MRI of the brain demonstrates abnormal frontal lobes bilaterally that include some degree of infarction, with abscesses and cerebritis. What is the pathogen?
Correct
Incorrect
Explanation:
The child is exhibiting signs of meningitis, as demonstrate by his lethargy and fever and confirmed by the elevated protein and decreased glucose in the CSF analysis. The MRI finding of abscesses is what determines the etiologic microorganism to be Citrobacter koseri, which is notorious for abscess formation in 80% of patients. This microorganism is a gram-negative enteric rod, resistant to ampicillin. The mode of transmission is most commonly vertical, from mother to infant during delivery. Treatment is usually a third- or fourth-generation cephalosporin and aminoglycosides for 4 to 6 weeks. The abscess area is frequently drained, with follow-up imaging to determine improvement.
Although Escherichia coli, Klebsiella pneumoniae, Listeria monocytogenes, and Streptococcus agalactiae (group B Streptococcus) can cause a brain abscess, Citrobacter is the most common bacteria causing the indolent course of meningitis described for a neonate with a brain abscess formation. -
Question 97 of 100
97. Question
1 pointsA 4-year-old boy with history of recurrent otitis media now has an eruption on his index finger. No systemic symptoms exist. The eruption has several open vesicles on his left second finger. His finger is very painful on palpation. What is the treatment?
Correct
Incorrect
Explanation:
This patient most likely has herpetic whitlow, a herpes infection at the terminal phalanx caused by herpes simplex virus 1 (HSV-1) and herpes simplex virus 2 (HSV-2). This painful infection of the hand most often involves the index finger or the thumb. In children, infections involving the fingers most often are caused by autoinoculation from primary oral HSV-1 lesions that are spread by finger or thumb sucking. The oral lesions, however, are not always present at the same time as the hand lesions because of the incubation period of the infection. The natural history of herpetic whitlow is to improve on its own in Iwo to two to weeks but many recommend treatment with oral acyclovir (400 mg TID) for ten days.
Although the efficacy of treatment has been unproven it is better to be on the safe side.
Incision and drainage is contraindicated in herpetic whitlow. Incision of the lesions results in delayed resolution that may lead to bacterial superinfection or systemic spread. Major complications may also occur, like herpes encephalitis. Hence, extreme caution should be taken.
Oral amoxicillin and topical bacitracin are not indicated in uncomplicated herpetic whitlow. Both could be indicated in cases of bacterial superinfection.
Topical ketoconazole is not used in herpetic whitlow. They are used for systemic and cutaneous fungal infections, respectively.
Cutaneous fungal infections usually are not associated with vesicular lesions. They are associated more classically with erythematous scaling eruptions. -
Question 98 of 100
98. Question
1 pointsA 4-year-old boy who has a ventriculoperitoneal shunt for congenital hydrocephalus develops fever, headache, irritability, lethargy, photophobia, and vomiting. He is up to date on his childhood immunization. His temperature is 39.6°C (103.2°F). He is noted to have nuchal rigidity, with the pressure of both Kernig´s and Brudzinski´s signs. The shunt tract is erythematous on the surface. A lumbar puncture is performed and shows WBC of 40,000/mm with 85% neutrophils, a glucose concentration of 48 mg/dL, and a protein concentration of 169mg/dL. Which of the following is the most likely pathogens?
Correct
Incorrect
Explanation:
Unlike meningitis occurring in normal children, ventriculoperitoneal shunt infections are most commonly caused by coagulase-negative Staphylococcus, such as Staphylococcus epidermidis. S. epidermidis causes 40%to 60% of all CSF infections in persons with ventriculoperitoneal shunts. Coagulase-negative Staphylococcus presents a significant threat to people who have indwelling devices or catheters. Most S. epidermidis isolates are resistant to multiple antibiotics, including nafcillin and oxacillin.
Vancomycin is the drug of choice to S. epidermidis infection. Removal of the indwelling medical device and parenteral anticiotic treatment are often necessary.
Kernig´s sign is positive if when the thigh is flexed at a right angle, complete extension of the leg is not possible, or causes exquisite pain. A positive Brudzinski´s sign is when bending of the neck causes flexion of the hips. Both signs are used to test nuchal-rigidity in evaluating patients for meningitis.
Haemophilus influenzae. Neisseria meningitidis, and Streptococcus pneumoniae are the most common pathogens of meningitis in normal hosts.
Pseudomonas aeruginosa is a rare cause of meningitis in children, with or without ventriculoperitoneal shunt.
Streptococcus Agalactiae, a group B streptococcus affects neonates. -
Question 99 of 100
99. Question
1 pointsA 5-year-old girl has a generalized vesicular rash and high fever since the previous day, and was ill the entire week. It looked like a couple of pimples on the childs face. The child has a history of Acute Lymphoblastic Leukemia. Temperature is 37.5°C (99.5°F), pulse is 90/min, blood pressure is 110/60 mmHg, and respirations are 16/mm. A total of 10 to 15 small, translucent vesicles on an erythematous base and 5 or 6 pustules, some with central umbilication, are present on the face, upper chest, and abdominal. Many small, erythematous macules and a couple of crusted lesions are seen interspersed between them. Which of the following is the most serious complication of the current condition that might occur in this patient?
Correct
Incorrect
Explanation:
This patient most likely has varicella (chickenpox), a primary infection with the varicella zoster virus vesicles, pustules, and crusts in various stages of evolution are the typical findings. Symptomatic treatment with oral rehydration, antipyretics, and topical antipruritic agents can be used. Aspirin is contraindicated with varicella because of the association with Reye syndrome. Varicella usually presents with cutaneous infection in immunocompetent hosts. However, in immunocompromised patients, such as this girl with acute lymphocytic leukemia (ALL), varicella can cause serious and life-threatening complications, such as varicella pneumonia varicella pneumonia has a very poor prognosis in patients with ALL, reaching a mortality of roughly 25%. The clinical course usually deteriorates rapidly and many patients die within 3 days of the diagnosis. Pneumonia is exceedingly rare in normal children, but it can happen in older adults.
IV acyclovir therapy is indicated in immunocompromised patients with varicella infection) Therapy should be initiated early in the disease course to maximize efficacy of the treatment varicella zoster immune globulin (VZIG) may modify the disease course if it is given shortly after the exposure. It is not useful, however, once the disease is established, as in this clinical vignette. Children with varicella also should not be given salicylates because of a theoretically increased risk of subsequent Reye syndrome. Patients should be both contact- and airborne-isolated during the entire course of the illness.
Other, less common, complications include encephalitis, hepatitis, glomerulonephritis, arthritis, and thrombocytopenia.
Cellulitis and endocarditis are not typical complications of varicella infection. -
Question 100 of 100
100. Question
1 pointsA 12-year-old has a changing spot of his scalp. The patient was born with a 1×1-cm bald spot near the crown of his head that enlarged as he grew. It always had a yellowish color to it and never grew hair. Recently, it became warty and more intensely yellow. Otherwise healthy, he has not taken any medication in the previous 6 months and has no known drug allergies. On inspection of the scalp there is a well-demarcated 2×3-cm, yellow, verrucous plaque with no hair. The surrounding scalp hair appears to be of normal quality and density. Which of the following is the most likely diagnosis?
Correct
Incorrect
Explanation:
This young man has a nevus sebaceous (of Jadassohn), a hamartoma that clinically presents as a sharply circumscribed, verrucous plaque varying from a few millimeters to several centimeters in diameter. This solitary lesion is always present at birth, most often near the vertex of the scalp, but also may be found on the face or neck, where it is usually linear in shape. It enlarges as the child grows and undergoes change at the onset of puberty when it starts to acquire a verrucous surface and increases in thickness with these changes, an increased risk of benign or malignant tumors developing within the nevus begins. Because malignancy may develop, the treatment of choice is excision at an early age.
Alopecia areata presents as rapid and complete loss of hair in one or, more often, several round or oval patches, usually in the scalp, beard area, eyebrows, or eyelashes. The patches are commonly between 1 and 5 cm in diameter and a few resting hairs may be found within them. There are no epidermal changes presents and the scalp skin appears completely normal except for the absence of hair. The lesions usually persist for several months before new hair growth occurs and, initially, fine, depigmented hairs will grow, to be subsequently replaced by normal hair.
Aplasia cutis congenita is a rare anomaly that presents as a congenital defect of the skin with a predilection for the midline of the vertex of the scalp. At birth a round or oval absence of skin and subcutaneous tissue is seen that subsequently heals from the periphery, and leaving a permanent atrophic and alopecic patch that persists unchanged thereafter. Very large or severe defects may require surgical intervention. Imaging studies of the head are indicated to rule out underlying defects of the cranium or neural tissue.
Inflammatory linear verrucous epidermal nevus is usually present at birth or appears in the first few years of life, and is mainly found on the extremities. Clinically, a linear, erythematous, verrucous, scaly or keratotic plaque that is intensely pruritic and resistant to topical or intralesional treatments is characteristic.
Sebaceous hyperplasia also known as senile sebaceous adenoma, is a common lesion found scattered over the face in the shape of small, cream- or yellow-colored umbilicated papules 2 to 6 mm in size. The age at onset is usually over 40 years. Treatment is for cosmetic purposes and may be done with electrodesiccation or fine electrofulguration. A shave biopsy should be done if there is any doubt about the diagnosis.